Copyright Outline

You might also like

Download as docx, pdf, or txt
Download as docx, pdf, or txt
You are on page 1of 76

Contents

Exam/General ......................................................................................................................................................... 3
INTRO/BACKGROUND ....................................................................................................................................... 4
COPYRIGHT VALIDITY (part 1) ......................................................................................................................... 6
Copyright Validity Attack Outline...................................................................................................... 6
Fixation ............................................................................................................................................................... 7
Originality ........................................................................................................................................................... 8
Other Non-Copyrightable Works ...................................................................................................................... 11
Derivative Works .............................................................................................................................................. 14
Derivative Work Validity Attack Outline ......................................................................................... 14
Compilations ..................................................................................................................................................... 16
Compilation Attack Outline .............................................................................................................. 16
Compilations of Non-Data ............................................................................................................................ 17
Useful Articles .................................................................................................................................................. 18
Pictorial, Graphic and Sculptural Works ...................................................................................................... 19
Useful Articles Attack Outline.......................................................................................................... 19
Computer Software ....................................................................................................................................... 23
AUTHORSHIP (part 2) ........................................................................................................................................ 26
Authorship Attack Outline ................................................................................................................ 26
Sole Authorship ................................................................................................................................................ 26
Joint Authorship ................................................................................................................................................ 27
Works Made for Hire ........................................................................................................................................ 28
FORMALITIES .................................................................................................................................................... 31
Duration ............................................................................................................................................................ 32
Duration Charts ................................................................................................................................. 32
Notes and Case .............................................................................................................................................. 33
INFRINGEMENT (part 3).................................................................................................................................... 36
Reproductive Right ........................................................................................................................................... 36
Reproductive Right Attack Outline .................................................................................................. 37
De Minimus Copying .................................................................................................................................... 38
Copying in Fact ............................................................................................................................................. 38
Illicit Copying ............................................................................................................................................... 40
Second Circuit Approach .......................................................................................................................... 41
Ninth Circuit Approach............................................................................................................................. 43
DERIVATIVE WORKS ....................................................................................................................................... 46
Derivative Work Right Attack Outline ............................................................................................. 46
1
Derivative Works With No Copying ................................................................................................................ 48
Characters ......................................................................................................................................................... 49
Characters Attack Outline ................................................................................................................. 49
Distribution Rights ............................................................................................................................................ 52
DISPLAY RIGHTS .............................................................................................................................................. 55
MUSIC .................................................................................................................................................................. 56
Sampling ........................................................................................................................................................... 57
Public Performance Right ................................................................................................................................. 58
FAIR USE (part 4) ................................................................................................................................................ 60
Fair Use Attack Outline .................................................................................................................... 61
Cultural Interchange.......................................................................................................................................... 63
Transformative Use ....................................................................................................................................... 65
Appropriation Art...................................................................................................................................... 68
Technology ....................................................................................................................................................... 70
Disassembly .................................................................................................................................................. 70
Online Search ................................................................................................................................................ 72
Access to Knowledge .................................................................................................................................... 73

2
Exam/General
*Copyright Act: https://www.law.cornell.edu/uscode/text/17

Exam (3,500 word limits)


 use the cases’ principles/argument in my arguments (don’t need to distinguish cases from one another)
o “this is like Feist because…”
 make arguments on both sides
 analyze each potentially copyrightable work and each potential infringement
o start from top: prime facie case for copyright validity: fixed, original…
o obviously spend more time on the complex issues
 ex: if it’s a photo, just say it’s a photo, so it’s fixed. Don’t go into lengthy discussion of
fixation.
 issues will be obvious, what matters is how you discuss the issues
 think about/write questions you would ask and why/how certain answers influence outcomes
o ex: in Batlin (Uncle Sam), we would ask Batlin why he made the choices he did. And a good answer
would be that “I changed the umbrella to make it look like my uncle’s umbrella, since I view Uncle
Sam as similar to my uncle.” A bad (for him) answer would be “oh it was too hard to emulate the
original.”
 don’t quote a statute/case unless interpreting a statute

Course Outline
1) Questions of copyright validity (who gets a copyright)
a. Has the author done enough/the right kind of stuff to merit a copyright?
b. Originality, creativity, fixed, 102(b), separability doctrine for useful articles
2) Details & technicalities of protectability, ownership, transfer of copyrights, authorship, duration
a. Nitty-gritty of copyright – how long they last, how to keep it going, etc.
3) Infringement & prima facie case – who you can sue once you have a copyright
a. What does the plaintiff have to prove in order to show that one of her rights has been violate
4) Fair Use (defense)
a. The effect of fair use on alleged infringement

***Circle: inside is original work, outside layer is derivative work (some transformation)
and outside circle is fair use (transformative)

3
INTRO/BACKGROUND
(1)

Property & Copyright Law


 in absence of real property law, ppl would just take stuff
 consumption is rivalrous and property goes into the hands of those who value it the most
 natural rights approach (not how we treat copyright law, but is the approach in many countries – preserving
the author’s moral/spiritual relationship w/things that he/she has created)
 but information is infinitely fungible (version of a song vs. another version of the song) unlike “stuff”
 consumption of information is non-depletable/non-rivalrous and ppl can consume this info simultaneously
 consider, in a world without copyright, the cost of purchasing a book would be competed down to the
marginal cost of physical reproduction.
o And if authors can’t make any money, they’re just going to stop writing books. This is where
copyright law steps in, to create incentives to create things
 Copyright law grants authors certain controls over the use of their works, including allowing them to charge
a particular cost for access to the work
o ex: our copyright book has a “copyright price” of $200

Why not let a copyright last for infinity years (more incentives to create)?
 This would negatively affect other ppl who may want to use the work
o if you want to use an existing copyrighted work, have to license/ask permission – obviously costs
$ if get permission)
 Would also negatively affect consumers who would then have to pay a higher cost for goods.
o Sometimes they can’t afford access; “deadweight loss”  someone who can’t afford a book so
doesn’t get access
 Copyright law attempts to balance the relationship between authors (who need financial incentives to
keep producing stuff) and consumers (who are interested in accessing work) and the general public
(concerns of freedom of speech, free expression) and downstream users (people who want to produce new
versions of existing works
o US copyright law balances these relationships as opposed to the moral/natural rights approach

Public Domain
 the reserve of all the stuff that’s not protected by copyright
 some bc copyright expired; some bc too generic; some bc came about before first copyright act (ex: R&J)

History
 copyright and patent law came about in same year but are very different areas of law
 copyright law from England, origin in Printers Guild attempting to restrain the behavior of rogue printers
o later, the Statute of Anne (England, 1709) made copyright law about authors’ interest (not
printers’)
 US: first Copyright Act in 1790: discusses formalities
 1909 Act: lengthens term of copyrights, adds more “things” that can be copyrighted (drawings, musical
compositions), lists these categories
 1976 Act is last major revision.
o Added new language to describe the sorts of works eligible for copyright protection
o No longer need formalities or the © -- copyright attaches once the work is created
o §101 is particularly important (definitions); often controls rulings
o Note: works created prior to the 1976 enactment are governed by the 1909 Act
4
IP Clause in Constitution:
“Congress shall have the power…To promote the Progress of Science and useful Arts, by securing for limited
Times to Authors and Inventors the exclusive Right to their respective Writings and Discoveries.”
 Read this statute using parallel construction (separating the patent and copyright sections)
o Copyright: “science” “authors” “writings”
 When Constitution written, “science” referred to knowledge and learning; clause
originally meant to protect maps, charts, books
o Patent: “useful arts” “inventors” “discoveries”
 “Promote the progress”: promote learning. This phrase is often seen as grounding the fact that US copyright
law takes a utilitarian approach (as opposed to moral rights approach)

5
COPYRIGHT VALIDITY (part 1)
(49)

Copyright Validity Attack Outline


1. Fixation per 102(a)
a. It is “by or under the author”?
i. Garcia: plaintiff didn’t fix it
b. More than transient?
i. Williams Electronics: audiovisual effects repetitive and limited by software
2. Originality per 102(a)
a. More than minimal creativity, not “typical” or “garden variety” (Feist)
i. ATC: assigning numbers to parts not minimally creative
ii. West: pagination isn’t copyrightable, totally random
b. Photos
i. Photographer orchestras the scene (Burrow-Giles, Mannion)
ii. View the photo as a whole, not just its individual components (Mannion)
1) Can’t copyright the non-protefctable elements of course (sky), but consider
them when viewing the photo & its copyright as a whole
iii. Original in 3 ways per Mannion
1) Rendition/technique: angle, light, background, composition
2) Timing
3) Creation of subject matter
iv. Even if goal of photo is just to depict the way something looks, can by more than shift in
medium due to artistic and technical choices (Schrock -derivative rights case)
c. Compilation:
i. Copyrighable if original in selection and arranagement (Feist) (see Compilation Attack)
d. Non-discimination principle: judges aren’t arbiters of taste (Bleistein – commercial art)
e. Has to be more than just a change in medium (Meshwerks)
f. Can consider intent (Meshwerks: trying to make exact copy; Schrock: trying to make photos look
different than the toys)
g. Filter out what’s not copyrighttable to determine what copyrightable expression remained (Skyy
Vodka – original contributions were copyrightable)
h. Note: will almost always find artistics works original, no matter how stupid they are
3. Other non-copyrightable works (102(b))
a. Cannot copyright systems, functions, processes, methods, etc.  maybe patent tho
i. Baker: functional/utility things not covered by copyright
ii. ATC: cannot copyright numbering system
b. Idea/expression distinction: cannot copyright ideas, only the expressions of ideas
c. Merger doctrine: where the idea and expression are so tied in together that there is only one (or
very limited) way to express something – the expression is not copyrightable
i. Baker: system & forms so intertwined, cannot use the system (idea) w/o the forms
(expression)
ii. ATC: only so many ways to express classifcations of parts (“big O rings” vs. “O rings”)
d. Thin copyright: where are more than a few ways to express an idea but scope of variation is
limited  scope of copyright is thin & only infringement if virtually identical copy
e. Scenes a Faire: cannot copyright incidents, characters or settings which, as a practical matter,
indispensable or standard in the treatment of a given topic

6
i. Hoehling: cannot copyright certain phrases and sequences (“Heil Hitler” and German
beer hall scene)
f. Facts: cannot copyright facts or theories of facts/interpretations (in public domain)
i. Hoehling: cannot copyright plot idea – interpretation of historical facts
ii. Compilations of facts: are copyrightable if creative enough (See Compilation Attack)

General Notes
 Scope: when assessing validity, you’re necessarily going to be discussing scope
o Consider: thin copyrights (Satava)

§102 of Copyright Act:


(a) Copyright protection subsists, in accordance with this title, in original works of authorship fixed in any
tangible medium of expression, now known or later developed, from which they can be perceived,
reproduced, or otherwise communicated, either directly or with the aid of a machine or device. Works of
authorship include the following categories:
(1) literary works;
(2) musical works, including any accompanying words;
(3) dramatic works, including any accompanying music;
(4) pantomimes and choreographic works;
(5) pictorial, graphic, and sculptural works;
(6) motion pictures and other audiovisual works;
(7) sound recordings; and
(8) architectural works.
^(a) tells us what’s protected
(b) In no case does copyright protection for an original work of authorship extend to any idea, procedure,
process, system, method of operation, concept, principle, or discovery, regardless of the form in which it is
described, explained, illustrated, or embodied in such work.
^(b) tells us what’s not protected

§ 202.1(a): Material Not Subject to Copyright


 Words and short phrases such as names, titles, and slogans; familiar sybos or designs; mere variations of
typographic ornamentation, lettering or coloring; mere listing of ingredients or contents;
 Ideas, plans, methods, systems, or devices, as distinguished from the particular manner in which they are
expressed or described in a writing;
 Blank forms, such as time cards, graph paper, account books, diaries, bank checks, scorecards, address
books, report forms, order forms and the like, which are designed for recording information and do not in
themselves convey information;
 Works consisting entirely of information that is common property containing no original authorship, such
as, for example: Standard calendars, height and weight charts, tape measures and rules, schedules or
sporting events, and lists or tables taken from public documents or other common sources.
 Typeface as typeface.

Fixation
§101 (definition of “fixed”)

7
A work is ‘fixed’ in a tangible medium of expression when its embodiment in a copy or phonorecord, by or
under the authority of the author, is sufficiently permanent or stable to permit it to be perceived, reproduced,
or otherwise communicated for a period of more than transitory duration. A work consisting of sounds, images,
or both, that are being transmitted, is ‘fixed’ for purposes of this title if a fixation of the work is being made
simultaneously with its transmission.

General
 rarely litigated but still important
 still fixed even if can’t “read it” (ex: paper in a player piano, computer software)
 Simple example of no fixation: if I recite an original poem and someone else writes it down and publishes it
– I have no copyright claim because I didn’t fix it
o but if I write it down (even if just write it down at home and no one ever sees it) – it’s fixed
 Snowman: is this sculptural work transient? Yes, but what if it’s in a really cold environment where it will
never melt?
 Garden: Per Kelley v. Millennium Park, gardens are not sufficiently fixed because they change over the
course of the season and are not stable
o Note that a rock garden would probably be deemed fixed

Different images on game fixed bc repetitive and limited by software


Video Game
Williams Electronics v. Artic (54) (1982)
F: P obtained 3 copyright registrations for his video game: (1) for the underlying software and then two for the
audiovisual effects during (2) “attract” and (3) “play” mode. D argues P has no copyright protection for the
audiovisual effects during “attract” and “play” modes because the effects aren’t fixed. Rather, they are too
transient because the game generates different images every time it is in one of these modes and involves/is
based on participation from the player.

H: The audiovisual effects copyrights are valid because the “works” are fixed. The audiovisual features repeat
themselves over and over when in the “play” and “attract” modes. Yes, the particular series of events has some
interaction from the player, but what the game can do is still fixed, and it’s basically the same thing every time.
While there’s some variation, it is structured/limited by the game’s existing commands and software.

Originality
(61)

General
 originality is not defined in §101
 law: originality requires some modicum of creativity. This is a low bar, but the creativity must be more than
typical, garden-variety. (Feist)
 copyright vs. patent law: the originality bar for copyrights is much lower than patents – why?
o Copyrights  taste is subjective – do some ppl prefer 20th century paintings to 21st century art? Sure!
Do some ppl prefer 1960s computers to current computers? No!)
 Facts: not copyrightable, but when facts are compiled in an original way, the work itself may be
copyrightable (Feist)
 copyright attached only to what is original to you
 effort: lots of effort may still not result in copyright (test is NOT “sweat of the brow”)

8
Photos are original – author’s mental conception – makes choices to express an idea
Oscar Wilde
Burrow-Giles Lithographic v. Sarony (1884) (61)
F: Photographer Sarony (P) filed copyright infringement suit against Burrow-Giles (D), who lithographed P’s
photo of Oscar Wilde and sold copies. Burrow-Giles argues that Sarony’s photo was not copyrightable arguing
that photos are not an “author’s writing” – but rather – the photographer just reproduces what’s in front of him
(no intellectual, creative ideas.)

H: Photos are copyrightable. “Writings” don’t just encompass books and maps. (Remember, photos didn’t exist
when wrote the Constitution!) An author is a person who has ideas and expresses them in a visible way. Photos
are how an author expresses an idea – the photo represents the mental conception of the photographer. Here,
Sarony made decisions about drapery, lighting, clothing of the subject, how to portray him, etc.  these choices
show authorship/originality.

Commercial purpose and bad art  still original


Circus Ads
Bleistein v. Donaldson (1903) (64)
F: Bleistein (P) hired by Circus to design circus ad posters. Later, Circus hired Donaldson (D) company to
make copies of P’s poster. P sued for copyright infringement and D argued that the ads P created were not
copyrightable bc not original.

H: Ads are copyrightable. Just because there’s a commercial purpose doesn’t mean not copyrightable. Judges
shouldn’t judge the aesthetic value of things (shouldn’t be arbiters of taste)  nondiscrimination principle.

Note: if they had no value, you wouldn’t have copied them!

Originality as low bar – adding a little to work in public domain


Mezzotint – Blue Boy
Alfred Bell & Co. v. Catalda (1951) (67)
F: Bell (P) created mezzotint reproductions of Blue Boy painting (OK bc in public domain). Catalda (D) made
copies of P’s work and sold them. Bell sued and Catalda argues that Bell’s mezzotints were not sufficiently
original to constitute a copyrightable work.

H: Bell’s mezzotints are copyrightable. The difference between the original (public domain) Blue Boy and
Bell’s mezzotints are attributable to Bell. A work doesn’t have to be strikingly unique or novel to get a
copyright. All an author has to do is contribute something “more than merely trivial” and that is “his own”.
Even if the goal of the mezzotint was to make an EXACT copy of Blue Boy, as long as there are some (even
accidental) variation, mezzotint would be copyrightable.

Note: If Catalda had copied the original painting, would have been fine (public domain) – but here, he copied
Bell’s work

Originality requires some minimal level of creativity, not “typical” or “garden-variety”


Phonebooks
Feist Publications v. Rural Telephone (73) (1991)
***most important case in contemporary copyright law; only case on originality since 1976 Act***
F: Rural (P) published phonebook which listed subscribers alphabetically in the white pages – it obtained this
info directly from subscribers. Feist (D) copied Rural’s listings and Rural sued for infringement. Note: Feist
9
originally contacted Rural and other phonebook companies to pay for the listings. Rural refused and Feist used
them anyway.

H: Rural’s phonebooks not sufficiently original. It is merely a compilation of facts (facts are not protectable).
And while compilations of facts can be protectable, here, the way the facts were compiled/arranged was not
sufficiently original. The selection of facts: names and phone numbers of people in the town; the
arrangement: by last name – simply lacks minimal creativity, is “typical” or “garden-variety.” All phonebooks
arrange names/phone numbers alphabetically.

**Important: while this case is about a fact compilation, the originality standard set forth applies to ALL works,
not just compilations

Think: should we be concerned about the lack of incentives to create things you can’t copyright, like
phonebooks? Tho, can monetize in other ways, like selling ads. Or could try to make it copyrightable by
organizing it in a certain way (ex: Westlaw key numbers) or just make ppl pay for access (again, Westlaw).

Photos are original to an author in 3 ways


Kevin Garnett photo
Mannion v. Coors (79) (2006)
F: Mannion (P) photographed Kevin Garnett in a particular way (white t-shirt, black cap, “bling bling”).
Defendant Coors began developing ideas for an outdoor billboard and used Mannion’s photo in a proposal.
Coors ended up not using the Mannion photo, but instead selected a similar photo (a black man wearing white
shirt, bling). Mannion sued.

Coors argues: Garnett made all these decisions, not Mannion and thus the photo is not original to the extent of
Garnett’s clothing, jewelry and pose.

H: Mannion’s photo is original/copyrightable. Court rejects Ds argument’s that Garnett made all the decisions --
Mannion orchestrated the scene – he photographed Garnett at a relatively unusual angle and used distinctive
lighting and composition (man against sky). He also instructed Garnett to wear simple/plain clothing and as
much jewelry as possible and told him to look “chilled out.” And the originality in the photo extends beyond the
individual clothing, jewelry and pose viewed in isolation. The entire image -- the man, sky, clothing and jewelry
in this particular arrangement -- is at issue, not just the individual components.

What’s NOT copyrightable


-Of course, Kevin Garnett’s face, torso and hands are not original to Mannion, and Mannion cannot prevent
others from photographing portraits of Garnett.
-Equally obvious, the existence of the cloudy sky is not original and Mannion cannot prevent others from using
a cloudy sky as a backdrop.

There are 3 ways in which a photo can be original to an author:


1) Rendition/technique/how shot the photo: angle of shot, light/shade, exposure, filters, effects achieved
by filters or developing technique, lenses, B&W vs. color, background, costume.
a. Altho these things aren’t independently original (ex: can’t prevent future photogs from using
sky as background!), the combo makes a photo copyrightable
b. Can protect the image but can’t prevent others from photographing same object or scene
c. Per Feist, we ask:
i. Do the choices owe themselves to the author?
ii. Modicum of creativity in the way object/subject is depicted?
10
2) Timing – ex: photo of salmon and brown bear
a. Can protect the image but cannot prevent others from photographing the same object or scene
i. Prof says this doesn’t really make sense  think about press conference where bunch of
photogs taking exact same photo
3) Creation of the subject matter – gathering/bringing together of a bunch of stuff in a particular
combination.
a. To the extent that a photo is original in the creation of the subject, an artist who
arranges/photographs a scene has the right to prevent others from duplicating that scene
b. Here: combination of Garnett, his pose, the bling, outfit, etc.

Wire models just copies, not original


Car Wire Models
Meshwerks v. Toyota (83) (2008)
**different than Bell – remember – this is post-Feist
F: Meshwerks took measurements of Toyota cars for ad campaign and created digital images resembling wire-
frame models.

H: Meshwerks models not original, were merely “very good” copies of Toyota’s cars. Nothing added, no
specific rendition, lighting, etc. Only difference was different medium. The models depict nothing more than
the un-adorned vehicles, which must be filtered out (Skyy Vodka), leaving no copyrightable matter. Intent also
matters, Meshwerks was trying to make copies of the vehicles. It wasn’t trying to create or add any original
expression.

Skyy Vodka: must filter out what is not copyrightable


Plaintiff photographer alleged that defendant photographer infringed on his commercial photographs of Skyy-
brand vodka. The court held that the vodka bottle, as a “utilitarian object”, a fact in the world, was not itself
copyrightable. The court did say that plaintiff’s photos reflected decisions regarding lighting, shading, angle,
background - and these original contributions were copyrightable. The defendant’s photos however, differed in
terms of angle, lighting, shadow, reflection and background, so it did not infringe on the plaintiff’s copyright.
The only constant between plaintiff’s and defendant’s photographs was the bottle, and an accurate portrayal of
the unadorned vodka bottle could not be copyrighted. Important: the vodka bottle had to be “filtered out” in
order to determine what copyrightable expression remained – because the vodka bottle did not owe its origins to
the photographers.

Other Non-Copyrightable Works


(90)

General
 102(b) tells us that certain items are excluded from receiving copyright protection: processes, procedures,
systems, methods of operation, concept, principle, discovery
o Statute seems to have two purposes: (1) defining the line between what is eligible for copyright
protection and what belongs in public domain; (2) define the line between copyrightable and
patentable subject matter
 Idea/express distinction: shorthand for the rule which requires the exclusion of a variety of elements of a
work from copyright protection – ideas are not copyrightable but the expression may be
o We don’t want to allow copyright protection for, say, the idea of doomed lovers whose families
oppose their marriage, or the scientific concepts embodies in Einstein’s theory of relativity
o This concept is simple enough, but difficult to implement it in practice, consider:

11
 A process is a series of steps. But isn’t sheet music merely “instructions” for playing a
piece? Sure, but doesn’t mean sheet music is a process!
 Think about function vs. things valued for their ornamental value (ex: yoga vs. dance)
 Merger doctrine: the idea and expression merge if they are so tied into together – there is only one (or very
limited ways) to express something – and then the expression is not copyrightable
 Thin copyright: where there are more than a few ways to express an idea, but the scope of variation is
limited. Here, courts may conclude that the particular expression is copyrightable, but that the scope of the
copyright is “thin” – so thin that infringement will only occur in the case of a virtually identical copy

Merger doctrine; Patent v. Copyright (expression vs. utility); Idea/Expression Dichotomy


Bookkeeping Book
Baker v. Selden (1879) (92)
F: Selden (P) obtained a copyright for his published book, which described the system of bookkeeping he
created, explained how to do bookkeeping and contained illustrated examples (blank forms/charts/tables) of
how to actually use/do the system. Baker (D) produced a very similar book with very similar illustrated
examples of his system. D clearly copied P’s book, and even improved on some of P’s methods. P sued for
copyright infringement.

H: P’s bookkeeping system/method is an idea and thus the method and illustrations are uncopyrightable. No one
but P has the right to print or public his book, but anyone can practice bookkeeping using his method and use
the illustrations P provided/included. The visual depiction of an invention doesn’t grant you protection for the
inventive components (if you write a book about how to make an airplane, doesn’t mean you own the rights to
make all airplanes). This case just seems more complicated because in describing the bookkeeping method, the
illustrations/diagrams correspond closely with the actual bookkeeping work (author is explaining and using the
system).

Merger doctrine discussion


-the system (idea) and forms (expression) are so intertwined that they merge
-the forms (expression) are incidental to the practice of the system – people cannot use the system (idea)
without the forms (expression), and since P cannot prevent people from using the system (idea), they merge

Patent v. Copyright discussion


-to protect P’s idea (invention), he should have gotten a patent – the court is cleaving off some stuff that
copyright law won’t touch – distinguishing between the subject matters of copyright and patent law
-court is distinguishing between the expression of bookkeeping and the function (or utility) of the bookkeeping
-expression vs. utility (not always an easy distinction) helps determine copyright vs. patent
-ex: painting is an expression. Sure, it has some utility, but main purpose is to appreciate how it
looks – its beauty
-ex: accounting forms are a utility. You appreciate it for its use/contribution – it’s about how the
forms work (not how they look)

Note: after this case, the only thing left of Selden’s copyright is the introductory literary explanation (if
someone copied it word-for-word)

Plot idea and literary devices (scenes a faire) not copyrightable


Hindenburg
A.A. Holdings v. Universal (1980) (98)

12
F: Hoehling published a non-fiction book about the Hindenburg disaster based on his research. His book
recounts the disaster, discusses investigations after the disaster and suggests that Spehl, a member of the crew,
was the saboteur. Later, Mooney published a non-fiction book about the disaster and Universal bought the
motion picture rights from him. Now, Hoehling sues Universal and Mooney for copyright infringement for
copying the plot of his book (particularly, his saboteur theory). Mooney admitted that he used Hoehling’s book
for certain details, but argues that Hoehling’s plot idea is not copyrightable.

Hoehling argument: Expression of my plot idea is copyrightable

H: The underlying plot idea and the way Hoehling described things (literary devices) not copyrightable.
1) Plot ideas: while expressions of ideas are copyrightable, interpretations of a historical event are not
copyrightable
a. we want to avoid a chilling effect on authors who contemplate tackling a historical issue or event
– giving them latitude to make use of earlier theories and plots
b. P’s plot idea is his interpretation of historical facts – this interpretation is not protectable
i. Facts are not copyrightable and neither are theories about the facts
- compare P’s book to compilation of facts (Feist)
2) Facts: facts in public domain, anyone can use the facts in P’s book
3) Scenes a Faire: reject P’s claim of infringement due to the random duplication of phrases and sequences
of events
a. Ex: all 3 works include a scene in a German beer hall, the phrase “Heil Hitler” or songs including
the German national anthem.
b. These elements are not copyrightable due to the scenes a faire doctrine: “incidents, characters or
settings which are as a practical matter indispensable, or at least standard, in the treatment of a
given topic”
c. It’s virtually impossible to write about a particular historical era or fictional theme without
employing certain “stock” elements or standard literary devises
d. If you’re writing about a French waiter, he’s probably going to be snooty! If you’re writing about
Germany, there’s going to be a beer hall scene, etc.

Policy: we don’t want the first person who goes through the archives and comes up w/ideas to have exclusive
rights to these facts and ideas – and then not allow future historians to analyze, change, write variations on
existing theories. Remember, progress in science! Think about the public benefit in encouraging the
development of historical and biographical works

Numbering system not copyrightable


Auto Parts
ATC Distribution Group v. Whatever It Takes Transmission (WITT) (105) (2005)
*how the law separates unprotectable systems/processes from copyrightable expressions*
F: ATC (P) sells transmission parts and published a parts catalog. WITT (D), a competitor, copied its catalog.
ATC claims that WITT infringed: (1) the individual part numbers contained in the catalog; (2) the illustrations
contained in the catalog; (3) the “Numbering System Manual.” WITT admits it copied but argues all of it was
uncopyrightable.

ATC argues: its catalog is a creative classification scheme or taxonomy and is copyrightable due these
creativities: (1) deciding what kind of info to convey in part numbers; (2) predicting future developments in the
transmission parts industry and deciding how many slots to leave open in a given sub-category to allow for
these developments; (3) deciding whether an apparently novel part that does not obviously fit in any of the
existing classifications should be assigned a new category of its own or placed in an existing category, and if so,
13
which one; (4) designing the parts numbers; (5) devising the overall taxonomy of part numbers that places the
parts into different categories

H: Not copyrightable.

Idea/expression dichotomy
 While classifications can be creative enough to satisfy the originality requirement, all of ATC’s “creative”
endeavors regarding its classification scheme are just ideas.
 ATC cannot copyright such ideas such as predicting how many types of sealing rings will be developed in
the future, its judgments that O-rings and sealing rings should form two separate categories of parts, its
judgment that a new part belongs w/the retainers as opposed to the pressure plates.

Merger doctrine
 Copyrighting the expression of ATC’s ideas about classification and future transmission are not barred by
the idea/express dichotomy, but rather the merger doctrine
 There are limited ways to express these ideas
 For almost all of the types of “creativity” claimed by ATC, there is only one reasonable way to express the
underlying idea
o Ex: only way to express the prediction that a max of 4 addt’l types of sealing ring might be
developed in the future is to leave 4 numbers unallocated
o Ex: “big O rings” vs “little O rings” vs “O rings”

Originality
 Copyrighting the allocation of numbers to sub-categories and parts is not barred by the merger doctrine but
it is barred due to lack of originality
 ATC merely attached certain numbers to parts, categories by a random process. These numbers simply serve
as useful shorthand way of referring to each part.
 No creativity in assigning random numbers

Policy: don’t want to let ppl get a copyright just by assigning a number – copyright law promotes progress of
science, doesn’t seek to reward the labor of authors

Derivative Works
(109)
(discussion of copyright validity -- fixation, originality, 102(b) in context of derivative works)

§101: Definition of “Derivative Work”


A “derivative work” is a work based upon one or more preexisting works, such as a translation, musical
arrangement, dramatization, fictionalization, motion picture version, sound recording, art reproduction,
abridgment, condensation, or any other form in which a work may be recast, transformed, or adapted. A
work consisting of editorial revisions, annotations, elaborations, or other modifications which, as a whole,
represent an original work of authorship, is a “derivative work”.

Derivative Work Validity Attack Outline


1. Per 101, was the work recast, transformed or adapted?
2. Minute variations that are functionally required added to otherwise public domain work not sufficient for
protection (L. Batlin)
3. If some originality involved, get copyright for the things you added (Schrock)
14
General
 the originality requirement for derivative works is the same as the originality requirement for other works –
being derivative just affects the scope of protection
o key inquiry: is there sufficient nontrivial expressive variation in the derivative work to make it
distinguishable from the underlying work in some meaningful way?
 to copyright a derivative work, it cannot infringe on the underlying work
 to create a derivative work, you need permission from the copyright holder of the underlying work (unless
public domain)

Small variations not sufficiently original


Uncle Sam Banks
L Batlin & Son v. Snyder (111) (1976)
F: Snyder created plastic Uncle Sam banks based on an original (public domain) iron bank. Batlin, a
competitor, seeks declaratory judgment that Snyder’s copyright is invalid.

Snyder argues: His banks are copyrightable due to their differences from the original. Differences: size, plastic
instead of iron, metal carpetbag is rough, metal bag fatter at base, eagle holds arrows, not leaves, size of Uncle
Sam’s face, etc. Also, relies on sweat-of-the brow arguments from Alva Studios (mini Rodin sculpture
copyrightable) by showing how difficult it was to produce his banks.

H: Snyder’s bank is not copyrightable. The small choices/variations are not sufficiently original (citing Bell).
And per Feist, sweat of brow doesn’t matter anymore.

Photos not derivative works, were original in their own right


Thomas & Friends Photos
Schrock v. Learning Curve (113) (2009) (photo as derivative work)
F: Learning Center hired Schrock to take product photos for promo materials. LC used the photos after the
usage restriction time period passed. Schrock sued, and LC argued that Schrock’s photos were merely
derivative works of the Thomas toys and that they were not sufficiently original for independent copyright
protection.

H: Schrock’s photos are copyrightable in their own right because they are sufficiently original – varying from
the underlying work. While the photos are accurate depictions of the toys, this is more than just a shift in
medium. Rather, Schrock’s artistic and technical choices combine to make a copyrightable photo that is his
own. The photos are original due to their originality in rendition (Mannion – lighting, camera, lens) – seen in
Schrock’s deposition when he talked about his creative process in depicting the toys – trying to make them look
“life-like, personable and friendly” and how he used various camera and lighting techniques to achieve this.
There is sufficient variation in angle, perspective, lighting and dimension to qualify the photos as
distinguishable from the underlying work.

Note: intent matters – Schrock was trying to create something different, whereas in Meshwerks, they were
trying to make an exact copy of the Toyota. Maybe the Meshwerks Ps could have argued they were trying to
present a different, new, cool version of the Toyota. And here, LC could have argued that their intent was for
Schrock to take photos of the toys that looked exactly like the toys.

15
Note: Altho LC argued they didn’t give Schrock permission to copyright his photos, once you create something,
you have the copyright (LC could have owned the copyright if put it in original K, but they didn’t).

Compilations
(121)
(discussion of copyright validity -- fixation, originality, 102(b) in context of compilations)

Compilation Attack Outline


1. Compilations of facts are copyrightable if the choices of selection and arranagmeent entail a minimal
degree of creativity (Feist) – remember that this copyright will be very thin, others can still use the facts
a. Feist: no originanlity, was typical to include names/phone numbers of ppl in town in alpha order
b. Maclean: creativity in choosing/arranging the data; also included editors prredictions
2. Merger Doctrine in Compilations
a. Opinion ideas don’t merge, fact ideas do (Maclean)
i. “Top 10 movies of the year” is an idea, but infused w/author’s opinion/taste
3. West Publishing: can’t copying ALL arranagements and re-arranagements made by manipulation of data
4. Compilations of Non-Data: look at the selection, arranagement and coordination
a. Total concept and feel – analyze as a whole (Roth)
i. Satava: thin copyright only for the original elements (NOT for everything else)

§101: Definition of “Compilation”


A “compilation” is a work formed by the collection and assembling of preexisting materials or of data that are
selected, coordinated, or arranged in such a way that the resulting work as a whole constitutes an original
work of authorship. The term “compilation” includes collective works.

Collective work: a work, such as a periodical issue, anthology, or encyclopedia, in which a number of
contributions, constituting separate and independent works in themselves, are assembled into a collective whole.

Factual compilations may be copyrightable – but not this one


Phonebooks
Feist Publications v. Rural Telephone (73) (1991)
See earlier for facts and holding

Authors of compilations typically choose which facts to include, what order to place them in, and how to
arrange the collected data so that it can be used effectively by readers. If these choices of selection and
arrangement entail a minimal degree of creativity, then they are copyrightable compilations of facts.
Remember, even if get a copyright on a factual compilation, this copyright will be very thin – others will be
able to use the facts.

Selection/arrangement of car values sufficiently original


Used Car Valuation
CCC Info Systems v. Maclean Hunter (122) (1995)
F: Maclean publishes Red Book eight times a year for three US regions, which sets forth editors’ projections of
the values for the next six weeks of “average” versions of most used cars sold in that region. These predicted
values are set forth separately for each car make, model number, body style and engine type. It also provides
predicted value adjustments for various options, and for mileage in 5,000 mile increments. These valuation

16
figures represent the editors’ predictions based on multiple information sources, as well as their professional
judgment. CCC provided Red Book’s valuation info through computer software and seeks a declaratory
judgment that it has incurred no liability to Maclean for taking and republishing Red Book’s info. Maclean
counterclaims, alleging infringement.

H: Red Book’s selection and arrangement of data is sufficiently original, and thus copyrightable.

Originality discussion:
 There was originality/creativity in the way Maclean selected, coordinated and arranged Red Book’s data.
o The editors chose arrange the data in a particular way (such as which years, models to include, to go
by region and not state or city, mileage increments at 5,000).
 The valuations not mere facts, but are predictions by the editors based on many data sources, professional
judgement and expertise.
o The editors are extrapolating to make judgements about the data (predictions of cost) – creative
decisions.

Note: Might be helpful to know what other competitors are doing – how are they arranging the data? What
mileage increments, etc.? Would help us know if what they did was really is original or just typical/garden-
variety like Feist’s phonebooks.

Idea/expression & merger doctrine discussion:


 CCC argues that Red Book just contains ideas: authors’ idea of the value of a particular vehicle; and to the
extent that expression is found in these valuations, these expressions are indispensable to the ideas (merger)
 While the contributions do consist of ideas, these are opinion ideas that do not merge, unlike factual ideas
o This issue was addressed in Kregos (P created form used to help predict outcomes of baseball games
by filling in nine stats of competing pitchers), which described two different categories of ideas:
 (1) ideas that explain phenomena or furnish solutions to problems (such as identifying
symptoms of a disease)
 (2) ideas, like the Kregos pitching form, that are infused with the author’s taste or opinion
o Red Book’s ideas are infused with the editors’ own particular beliefs/judgments, whereas someone
else may have an entirely different interpretation (“top 10 movies of the year”)
o Every compilation will involve ideas from the author, such as what data to include and how to
arrange it. If these ideas merged with the expressions, then there would be no point to copyrights in
compilations

Westlaw Page Numbers


Matthew Bender & Co v. West Publishing (126) (1999) (applying compilation analysis to non-compilations)
F: West creates and publishes compilations of judicial opinions. Bender does the same, selecting and arranging
the cases differently, but informs readers on which page the West Reporter version of the case starts (West’s
pagination). West sues Bender for copyright infringement, arguing that copying its pagination allows Bender
users to "perceive" West's copyright-protected arrangement of cases.

H: West’s pagination is not copyrightable, there is no creativity or originality in the pagination, it’s totally
random (like ATC). The pagination also does not infringe on West’s arrangement of cases. To agree with
West’s version of “copy” would expand a copyright to include ALL arrangements and re-arrangements made by
the manipulation of data.

Compilations of Non-Data
17
*selection, arrangement and coordination of elements other than data

Cards original in their total concept and feel


Greeting cards
Roth Greeting Cards v. United Card Co. (131) (1970)
F: Roth claims United copied seven of its greeting cards.

H: Roth’s cards are copyrightable. Here, the proper analysis of copyrightability was the total concept and feel
test: must analyze ALL of the cards elements as a whole: text, arrangement of text, art, characters, mood
portrayed and association between art work). We consider the cards like a compilation of its textual and visual
materials -- can’t just analyze individual parts.

Think: We look at the forest, not the trees, but also must ignore all the “unoriginal trees” – the uncopyrightable
aspects (here, the words). This is like the Skyy Vodka cases, where we ignored the unoriginal aspect – the bottle.

Note: If say, United had only copied the little bits of text, because that was not independently copyrightable,
that would have been OK.

Scope of the copyright: thin copyright for the original elements only
Jellyfish Sculptures
Satava v. Lowry (133) (2003)
F: Satava made jellyfish sculptures. Lowry made similar jellyfish sculptures and P sued for infringement.

H: Satava does not have copyright protection for the idea of producing a glass-in-glass sculpture, or for the
elements of expression that naturally follow from the idea of such a sculpture (such as depicting jellyfish with
tendril-like tentacles or depicting jellyfish in bright colors). But Satava may prevent others from copying the
original features he contributed – a thin copyright.

Discussion
 Combinations of individually unprotectable elements may qualify for copyright protection, but not all
combinations will qualify
o A combination of unprotectable elements is eligible for copyright protection only if those elements
are numerous enough and their selection and arrangement are original enough that the combination
constitutes an original work of authorship
 Here, the combination of unprotectable elements fall short of this standard ^
o The selection of clear glass, oblong shroud, bright colors, vertical orientation, and stereotyped
jellyfish form, considered together, lacks the quantum of originality required.
o These elements are so “commonplace” in glass-in-glass sculptures and so typical of jellyfish
physiology – if the court were to grant Satava a copyright, this would effectively give Satava a
monopoly on glass-in-glass sculptures of vertical, single jellyfish.
o Consider that if you’re going to depict jellyfish in glass, it’s going to look like this
 Thin copyright: Satava has added some copyrightable elements  distinctive curls of particular tendrils,
arrangement of certain hues, unique shape of jellyfish bells. He has a thin copyright on these aspects

Note: Stronger argument for Satava here would be to show/discuss all of the options he had and then why and
how he made all these little decisions

Useful Articles
18
(177)

Pictorial, Graphic and Sculptural Works


**remember, these works still have to be original and fixed to get © protection. This is just an additional bar we
require for PGS works

Useful Articles Attack Outline


1. Is this a PGS work under §101?
a. If NO  N/A
b. If YES  is it a useful article? See §101; (think: it is useful? Does it have an intrinsic utilitarian
function?) (ex: Pivot mannequin head is useful article)
i. If NO  N/A
ii. If YES  to determine if the work is copyrightable, ask if it has aesthetic features that are
physically or conceptually separable from the utilitarian aspects of the work.
(1) Think about how the item came about/was commission; why was it created and why
in this way (try to justify aspects of the design)
(2) Separable/copyrightable
- PGS features exist independently of the functional aspects (Mazer)
- PGS elements reflected the designer’s artistic judgment, was independent of
functional/utilitarian concerns and influences (Pivot)
- artistic features primary, utilitarian features secondary (Pearl)
- designs are not necessary to the utilitarian functions of covering body,
wicking moisture and permitting free movement (Varsity)
(3) Not separable/not copyrightable
- Utility and aesthetic features inextricably intertwined (Carol Barnhart)
- Design elements reflect utilitarian considerations only (Brandir)

General
 copyright law imposes a special doctrinal mechanism for determining whether functional yet creative works
are copyrightable
 these cases show how copyright law deals with works that are both ornamental/aesthetic and
functional/utilitarian
 law attempting to separate the realm of copyrightable subject matter from patentable inventions
o underlying concern is that some works are too functional for copyright law

§101: Useful Articles Definition (codifies Mazer)


Useful Article:
 an article having an intrinsic utilitarian function that is not merely to portray the appearance of the
article or convey information. An article that is normally a part of a useful article is considered a “useful
article”.

Pictorial, graphic and sculptural works (PGS):


 include two-dimensional and three-dimensional works of fine, graphic, and applied art, photographs, prints
and art reproductions, maps, globes, charts, diagrams, models, and technical drawings, including
architectural plans. Such works shall include works of artistic craftsmanship insofar as their form but not
their mechanical or utilitarian aspects are concerned; the design of a useful article, as defined in this
section, shall be considered a pictorial, graphic, or sculptural work only if, and only to the extent that,
19
such design incorporates pictorial, graphic, or sculptural features that can be identified separately
from, and are capable of existing independently of, the utilitarian aspects of the article.

Examples
Remember, just because something is useful for some other purpose than its aesthetic doesn’t mean it’s a useful
article. And just because a useful article is “pretty” doesn’t mean it gets copyright protection.
 Airplane  useful article
 Toy airplane  not a useful article. Its value is entertainment and imagination; “playing” or providing
pleasure is not a use.
 Costume  likely a useful article. Its intrinsic value is it “be something else”
 Mask  likely not a useful article. Not intrinsically functional.
 Car  useful article
 Painting  not a useful article. Its value is it portray its own appearance. When you look at it, you don’t
think about what you can do with it. It may be useful for selling or to cover a whole in the wall – but it is not
a useful article.
 Apple computers  useful article, but no copyright protection (altho is pretty!)
 Heart-shaped spoon  useful article, but no copyright protection bc the “cute” aspect not separable from the
functional aspects
 Jaguar hood ornament  useful article and copyright protection. You can take the ornament off and the
car still works
 Ornamental carving on chair  useful article and copyright protection. You cannot take the carving off the
chair – but it exists independently from the underlying functionality of the chair

Physical separable  copyrightable


Lamp Sculpture
Mazer v. Stein (1954) (178)
F: Mazer made sculptures in the form of human figures. Later, he added lamp components to the sculpture –
making the sculpture a base for lamps. Stein created similar lamps and argued that Mazer’s sculptures were no
longer copyrightable because they were lamp bases. And lamps are not art, but are items of industrial design,
and covered by patent and not copyright law.

H: Lamp base is a PGS work (sculpture), useful article and copyrightable because the sculptural feature can be
physically separated/exist independently from the functional aspects of the lamp. Just because it’s an article of
industrial design, doesn’t mean it can’t be copyrighted. As long as there are still aspects of the work that qualify
it as a work of art, then it is still potentially copyrightable. It’s OK if the copyright system and design patent
system occasionally overlap.

Conceptually separable  copyrightable


Mannequin head
Pivot Point v. Charlene Production (2004) (181)
F: P sold “Mara” mannequins with a “hungry look,” for makeup and hair design. Ds copied the sculpture.

H: Mannequin head is a PGS work (sculpture), useful article and copyrightable.

Useful article: head made a certain way in order to be functional as hair/makeup model (Picasso head wouldn’t
have been good)
20
Conceptually separable features: mannequin head has conceptually separable features which can be identified
separately from the functional aspects of the head.
 Applied the Brandir test: design elements that reflect the designer’s artistic judgment, independent of
functional influences/utilitarian concerns = conceptual separability.
o The mannequin head artist didn’t have specifications/measurements from her employer about what
to create – she was just told to create a “hungry” look – then went off to do that without concerning
self with any utilitarian or functional concerns.
 Consider that there are lots of ways an artist could make a mannequin head look “hungry” but unlike Mara.
This variation of choice matters – usually when something is functional we think of there being only one
way for it to function.
o Court isn’t not worried about the anti-competitive effects of this case bc there are lots of other ways
designers can design mannequin faces and still look hungry

Court discusses several other cases


 Kieselstein-Cord v. Pearl: jeweled belt buckles were useful articles with PGS elements which were
conceptually separable  copyrightable.
o evidence of buckles being used in non-buckle ways (as necklace); in permanent collection at MoMA
o artistic features are primary, the utilitarian features are secondary

 Carol Barnhart v. Economy Cover: human torso mannequins (displaying shirts/jackets) were useful
article with PGS elements  not copyrightable
o There were no aesthetic/artistic features which were separable from the forms’ use as utilitarian
objects to display clothes.
o Whereas the Pearl buckles’ sculptural features were superimposed onto the buckles, here, the torso
form was required by the function. Its utility was “inextricable intertwined” with the form.
o This holding doesn’t mean the torsos weren’t aesthetic – just don’t get a copyright

 Brandir v. Cascade: wire bike racks were useful articles with PGS elements  not copyrightable
o There were no aesthetic elements which were separable from its use as an useful article
o The racks are beautiful marriage of form and function, but too close of a marriage
o The design elements don’t reflect the unconstrained perspective of the artist or designer’s artistic
judgment independent of utilitarian considerations
o When it was just a swirly sculpture – likely copyrightable, but once made it higher, bolted to ground
etc. in order to serve a utilitarian purpose…no longer copyrightable
 The combo of aesthetic and functional characters are inextricably intertwined in the work
 The changes made to the sculpture were informed by functional concerns

Graphic designs on uniforms are separable, and thus copyrightable


Cheerleading Uniforms
Varsity Brands v. Star Athletica (separate) (6th Circuit)
F: Varsity designs and manufactures cheerleader uniforms. Its designs include graphical elements including
stripes, chevrons, zigzags and colorblocks. Star Athletica sold cheerleading uniforms that were similar in
appearance to Varsity’s and Varsity sued for copyright infringement.

Star Athletica argues: Varsity’s uniform designs are not conceptually separable from the functionality of the
uniforms – and are thus not copyrightable.

21
Varsity argues: The uniform designs are conceptually separable – they exist in a tangible medium other than on
the uniforms.

H: Varsity’s uniform designs are copyrightable. The PGS features of the graphic designs are conceptually
separable from its utilitarian functionality.

Analysis of Varsity’s uniform designs


1) PGS work? Yes, they are two-dimensional works of graphic art
2) Are the designs a useful article? Yes.
a. The designs are sketches that depict cheerleading tops and skirts – the components of a
cheerleading uniform. In other words, they are designs of cheerleading uniforms/sportswear which
have an intrinsic utilitarian function that is not merely to portray the appearance of clothing or to
convey information.
3) What are the utilitarian aspects of cheerleading uniforms?
a. Sweat-wicking, movement, modesty
b. *Rejects Star’s argument that fact that uniforms only identify an individual (definition of useful
article – convey information) as a cheerleader
4) Can viewer identify the PGS features separately from the utilitarian aspects? Yes
a. *Rejects Stars argument that the PGS features are inextricable intertwined with the utilitarian
aspects of the uniform because they serve a decorative function
i. Agreeing with Star would result in nearly all artwork being unprotectable! A Mondrian would
be unprotectable bc it decorates the room in which it hangs. And it would make fabric
designs, which makes garments more attractive, ineligible for copyright protection. And of
course paintings and fabric design are copyrightable.
ii. So, just bc a PGS work has a decorative function, doesn’t mean it’s not separable from the
utilitarian aspects of the article
5) Can the PGS features exist independently of the utilitarian aspects of a cheerleading uniform? Yes
a. Consider if the PGS features are required by the article’s utilitarian function, consider what was
the designer thinking
b. The color blocks, zigzags, etc. are wholly unnecessary garment’s ability to cover the body, permit
free movement and wick moisture.
c. Varsity’s designers sketch the designs and then select, place and arrange various graphic elements.
Then the production dept either applies these graphic designs onto uniforms or create the designs
by sewing panels of fabric together. Then the designs may be incorporated onto the surface of
different garments including warm-ups and jackets
i. Evidence establishes that the designs are transferrable to articles other than the traditional
cheerleading uniform
ii. Interchangeability of the various Varsity designs (picking different colors, styles) is evidence
that the graphic design on the surface of the uniform does not affect where the uniform still
functions as a cheerleading uniform
iii. Nothing (save perhaps good taste) prevents Varsity from printing or paintings its designs,
framing them, and hanging the resulting prints on the wall as art

Dissent: There's no conceptual separability, the designs not copyrightable. Like Jovani (prom dress where court
excluded the ruching, tulle lawyers, and arrangement of sequins from the copyright), the designers’ aesthetic
considerations merged with the functional concerns to cover the body and identify the wearer as a member of
the squad. Without the graphic designs, Varsity’s designs would lose their ability to identify the wearer as a
cheerleader. One of the main functions of the uniform is it identify the wearer as a member of a team.
The placement of the stripes, chevrons etc. is not separable from the function.

22
Note: copyright protects fabric designs but not dress design (ex: v-neck; a-line skirt shape) – because cannot
separate the “dress design” (aesthetic features) from the utilitarian aspects of the clothing – to cover, protect and
warm the body

Computer Software
(206)

§101 Definitions
Computer program: a set of statements or instructions to be used directly or indirectly in a computer in order
to bring about a certain result.

Literary works: works, other than audiovisual works, expressed in words, numbers, or other verbal or
numerical symbols or indicia, regardless of the nature of the material objects, such as books, periodicals,
manuscripts, phonorecords, film, tapes, disks, or cards, in which they are embodied.

History
Congress formed a taskforce after 1976 Act (CONTU) to figure out the proper treatment of computer software.
CONTU often resorts to analogies, analogizing old material to the new – often understanding computer
software as a literary work (or videotape, symphony). A literary work is considered a set of symbols in a
medium, and a code considered a writing.

Programming basics
 Source Code: written by and read by humans (programmers language)
o run through a translating program into object code, so that a computer can run the operation
o people use can different source codes and produce the same output
 Object Code: read only by computers (computer language)
o software companies put their codes out into the world in this form
o translation of otherwise readable source code
 HTML, Java etc. are compilers that turn Source Code into Object Code
 Operating system: underlying mechanism which tells the hardware what to do
 Applications: these interact with the operating system
 Computer programs: a set of statements or instructions used directly or indirectly in a computer in order to
bring about a certain result

Object code copyrightable


Apple-Combatable Programs
Apple v. Franklin: (208) (1984)
F: Franklin copied Apple programs in order to make programs that would be compatible with Apple’s operating
system/interface. Franklin argues that the OS not subject to © bc it’s a method or process – instructing the
computer to do something (definition of computer software in the Act  “set of instructions to bring about a
given result”).

H: Apple’s OS, source and object code are all protected by copyright. As a matter of copyright validity,
copyright protects aspects of both the source and object code to the extent that there is some creative
expression.
 altho software has functional goals, it may still be a writing (a copyrightable work)
 while the value of software resides in its functionality, there may be a degree of expressiveness to it
o aesthetic choices of programmer (the way programmer decides to write the code) can be beautiful or
creative, even “symphonic”
23
o expression opportunities in the manner in which the programmer decides which codes to use
 also, is important for companies to be able to protect object code too
o just protecting source code wouldn’t be very helpful bc ppl could use different source code and still
produce the same output
o basically, if ppl can copy object code, can copy the program

Merger discussion: has the idea (what the program does) and the expression (the code that causes the computer
to run the program) merged?
-no, they have not merged because individuals can use different language (source code) to express the same idea
(result) idea
-the court is not going to look at the actual software (the expression) because they don’t understand it, rather,
they’re going to ask questions to determine if there are multiple ways of expression (and here, there is!)

After this case


-Computer software plaintiffs begin to push a little further on this literary works analogy
-plaintiffs are trying to expand beyond “literal infringement” (copying the code) to “non-literal copying” 
copying the broader organization of the code

Program structure copyrightable


Non-literal copying
Computer Associates v. Altai (212) (1992)
***use the abstraction-filtration comparison test from Nichols – taking out all of the functional stuff first
F: P made a software and D copied aspects of the code, P sued for infringement and won. D went back and
rewrote the code so that D’s code doesn’t look like P’s. P claiming still infringement on the non-literal
arrangement of the code itself (the general arrangement of the modules and sub-modules within the code – the
structure).

I: Can an author get a copyright not only for literal code, but also for the broader arrangements of the code –
making an analogy to protecting literary works where we sometimes grant copyrights for not just the words, but
also for the plot/overarching structure.

H: Yes, copyright protection extends to non-literal arrangement of the code.

Abstraction-Filtration-Comparison test (Nichols)


-use this test to determine if infringement at some higher level – looking at overall structure
(1) Abstraction: take the work and conceptualize it at different levels of generality
o break it down and think about the different levels, from most general to most specific, and analyze
o range: particular line of code in module giving individual instruction  function of the module 
function of high level modules  ultimate function of the software program
 example, Romeo & Juliet: general  play about star-crossed lovers; specific the actual
words
(2) Filtration: filter out the uncopyrightable stuff
o efficiency: take out the aspects of the design that are functional elements dictated by efficiency
 ask whether a given aspect of a program’s structure is necessary to efficiently implement part of
the process. If yes, then that choice has merged with the underlying function of the module.
o external factors: take out the scenes a faire of computer programming.
 the mechanical specifications, design standards, widely-accepted programming practices,
compatibility requirements, demands of the industry
o public domain: take out any elements from the public domain
24
(3) Comparison: once all of the noncopyrightable elements have been sifted out, what remains is the
copyrightable “golden nugget”

Small bits of code (APIs) copyrightable


Java
Oracle (Java) v. Google (229) (2014)
**Application programming Interfaces (APIs) are small bits of code**
F: Google developed Android platform with API packages – some of which copied Java APIs. Essentially,
Google copied the particular commands that are typed into the Java code to tell the virtual machine what to do.
Virtual machines are sets of programming routes that interact directly with the specific hardware/software of a
device. Oracle sued Google for copyright infringement.

Note: APIs define doorways to the various preexisting functions contained in the virtual machine; also define
doorways to reusable blocks of code written once to perform particular functions and then incorporated into
many products. Sometimes its available open source, sometimes licensed by the original author

I: Are these APIs (small bits of code) protectable?

H: Yes, there is a valid copyright in these short bits of code.


 not a method/system, but an expression of a method
o expressions or descriptions of methods are copyrightable as long as some degree of
originality/creativity -- here there is creativity
 no merger: there are multiple ways the coders could have written the code

25
AUTHORSHIP (part 2)
(137-153; 169-172)

Authorship Attack Outline


1. Author has control (Titanic, Lee)
2. Joint Authors: work prepared by 2 or more authors w/the intention that their contributions be merged into
inseparable or interdependent parts of a unitary whole
a. All contributions must be independently copyrightable (Trinity)
b. Must be intention for someone to be a joint author (Lee)
c. No copyright protection for individual performers in a work (Garcia)
3. Works Made for Hire
a. (1) applies to works prepared by an employee within the scope of his employment
b. (2) works commission/ordered/contracted (e.g., contributors to movies)
i. Doesn’t apply to photos, sound recordings, sculptures
ii. CCNV: used common-law agency principles to determine if a “freelancer” was actually an
employee

§201: Ownership of Copyright


Initial Ownership: Copyright in a work protected under this title vests initially in the author or authors of the
work. The authors of a joint work are co-owners of copyright in the work.

Works Made for Hire: In the case of a work made for hire, the employer or other person for whom the work
was prepared is considered the author for purposes of this title, and, unless the parties have expressly agreed
otherwise in a written instrument signed by them, owns all of the rights comprised in the copyright.
^Copyright Act doesn’t tell us when someone is an author of a work

General
 authorship is all about who should be entitled to claim copyright protection
 we’re discussing is default rules – rules that govern the copyrighted works if rules not stated by K
 specifically, Garcia and Lee help us think about the relationship and intent among the parties
 if multiple people have contributed to a work, look for joint works or made for hire

Sole Authorship
(138)

Titanic
Lindsay v. The Wrecked and Abandoned Vessel R.M.S. Titanic (138) (1999)
F: Lindsay is a filmmaker/director – shot a movie about Titanic’s wreckage using special cameras and lights.
He never actually went down to the wreckage and didn’t actually operate the camera.

H: Lindsay is the author. The divers are down there carrying out his instructions, ex: “shoot from this angle” –
he conceived what the divers should do. He had an independent mental conception (Burrow Giles) and
controlled the filming.

Monkey Selfie
-Artist Slater “got” a monkey to take a selfie, published the photo and claimed copyright. Who’s the author –
Slater, monkey, no one?
-Consider, how much contribution does one need to make in order to be an author – giving the monkey the
camera enough? Did Slater know what the monkey was going to do?
26
Joint Authorship
(140)

§101: Definition of “Joint Work”


Joint work: a work prepared by two or more authors with the intention that their contributions be merged into
inseparable or interdependent parts of a unitary whole.

General
 Joint authors get a pro rata share in the work regardless of the extent of their contribution
 If exclusively license a work, need consent from all the joint authors
 If non-exclusively license a work, just need to account to the other joint authors for any income from such
license (don’t need their permission)

Actors’ contributions not independently copyrightable  not joint authors


Actors’ suggestions
Erickson v. Trinity Theater (140)
F: While working at Trinity Theater, Erickson wrote two plays. While developing them, she conducted
improvisational idea sessions with Trinity actors. Some of the actors’ suggestions during these sessions were
adopted into the completed scripts, though Erickson retained final control over what material was chosen.

H: Trinity actors not joint authors of the plays.


 Court applied Goldstein test, which requires that each of author’s contribution be independently
copyrightable (sufficiently creative, original, etc.) Trinity Theater could not identify any copyrightable
contributions made by the actors.
o If actors had contributed say, a particular section of dialogue, then would have to decide if
independently copyrightable and go through the whole Feist analysis to determine if joint authors
 Considers applying but rejects the Nimmer de minimus test, which requires that only the combined product
of joint efforts be copyrightable (even lower than Goldstein test)
 Policy: everyone who gives advice/input should not be considered an author – would have to have
everyone who ever gave advice sign a contract stating they’re not an author (or would all have claims)

Note: Goldstein test is still a fairly low standard, could cause problems in collective works when contributors do
something that is minimally creative/original

Never intended P to be joint author  not a joint author


Malcolm X
Aalmuhammed v. Lee (141)
**This case adds another element to the joint author test  intent
F: Spike Lee co-wrote, directed, and co-produced Malcolm X. Aalmuhammed (JA) knew a lot about Malcolm
X’s life and Islam, which played an important role in the movie. Under Lee’s supervision, JA assisted with
production of the movie, including directing the actor playing Malcolm X, suggesting script edits that were used
in the final cut, ensuring historical and religious accuracy and editing some of the movie in post-production. Lee
had final say on all of JA’s contributions. JA didn’t have a contract w/ Lee or Warner Brothers. JA requested
credit as cowriter and was denied, credited as “Islamic Technical Consultant.” He sued Lee, WB and Lee’s
production companies, seeking declaratory ruling that Malcolm X was a joint work, and that he was joint author.

H: Not a joint work – JA not an author.


 authorship is not the same as simply making a valuable and copyrightable contribution (which JA did)
27
 author is traditionally meant to refer to the originator of the work – the person who causes something to
come into being – not JA. Warner Bros and Spike Lee had control
 if authorship was about making a creative substantial contribution, then everyone from the hairstylist to the
casting director would be an author
 altho JA’s contributions independently copyrightable and were intended to be a part of the work (Erickson)
there was no intent for JA to be a joint author. Court seems to be reading statute to read that there cannot
just be an intent to merge works, but there must also be intent to be joint authors.

Policy: progress would be retarded and not promoted if an author could not consult with others and adopt their
useful suggestions without sacrificing sole ownership of the work

Note: If JA had had work-for-hire K like everyone else on the movie, this wouldn’t have been a problem!

Rent
Thomson v. Larson
No evidence that dramaturg and author ever intended the dramaturg to share credit as author – never treated or
billed as author. Dramaturg also had no decision-making authority. Larson was sole author.

Actress not joint author of her 5-second performance in trailer


Anti-Islam Film
Garcia v. Google (separate)
F: Garcia “tricked” into acting in anti-Islam film. Movie trailer on YouTube (owned by Google) – linked to
violent protests, she got death threats. She sued Google, arguing that she was author of her 5-second
performance in trailer, and that she owns the copyright to it.

H: She’s not the author of her scene, there’s no copyright protection for an individual performer in a work.
Individual performances EW merely contributions to the overarching work – the movie. Also, she didn’t “fix”
the work herself, as required by the statute (“a work is fixed…by or under the authority of the author”) -- it was
“fixed” by the director and his crew. Plus, policy  to give her protection would splinter a movie into many
different “works”. Would enable any contributor from a costume designer to an extra to claim a copyright in the
random bits and pieces of a movie.

Works Made for Hire


(146)

§101: Work Made for Hire


 (1) a work prepared by an employee within the scope of his or her employment; or
 (2) a work specially ordered or commissioned for use as a contribution to a collective work, as a part of a
motion picture or other audiovisual work, as a translation, as a supplementary work, as a compilation, as an
instructional text, as a test, as answer material for a test, or as an atlas, if the parties expressly agree in a
written instrument signed by them that the work shall be considered a work made for hire.
^note that sound recordings, photographs, sculptures (CCNV) not included in (2); cannot be contracted into
a work for hire
o Supplementary work: a work prepared for publication as a secondary adjunct to a work by another
author for the purpose of introducing, concluding, illustrating, explaining, revising, commenting
upon, or assisting in the use of the other work, such as forewords, afterwords, pictorial illustrations,
maps, charts, tables, editorial notes, musical arrangements, answer material for tests, bibliographies,
appendixes, and indexes

28
o Instructional text: a literary, pictorial, or graphic work prepared for publication and with the
purpose of use in systematic instructional activities.

General
 Issue: copyright relationship where A (has no copyright interest) creates work for B (owner)
 two ways this comes about:
o 1) when you’re an employee and work is done in the scope of your employment
 ex: law firm asks you to write a brief, employer owns copyright to the brief
o 2) a work specially ordered or commissioned
-this only applies to specific classes of works (only the works listed in (2))
-ex: Lee  all the other ppl who signed work-made-for-hire agreements – these were valid
agreements bc one of the categories listed is motion picture
-photographs, sound recordings not on the list – can’t be contracted into a work made for hire

Note on Assignments and WFH


 Assignments are terminable in copyright law and are terminable after 35 years (can change your mind)
 WFH has no termination rights

Not an employer/employee relationship  not work-for-hire


Homeless sculpture
Community for Creative Non-Violence v. Reid (147)
**note: sculpture isn’t covered under (2) so only looking at (1)
F: CCNV hired Reid to create sculpture about plight of the homeless. At CCNV’s request, Reid submitted
several sketches of the statue for approval. Consultations between Reid and CCNV led to a sculpture that was
significantly different from what Reid had originally conceived. On several occasions, a CCNV representative
checked on progress and to coordinate CCNV’s construction of the sculpture’s base. After initial displaying,
CCNV wanted to take statute on tour of cities, Reid objected and refused to return it. CCNV sued.

H: Not work for hire -- Reid was an independent contractor, not an employee who created sculpture under the
scope of employment.
 paying and giving instructions not sufficient to categorize as an employee, and Congress only intended
WFH to cover works under employee-employer relationship
 the court discusses different “control” tests to interpret “employee” within Copyright Act, picks one
o right to control: if hiring party has right to control behavior of hired party, the hired party is an
employee (which was the case here). Court rejects as too broad, not what Congress intended. Too
similar to part (1) of statute, would make rest of statute redundant
o actual control: wielded control of the party. Court rejects as too broad, not what Congress
intended. Too similar to part (1) of statute, would make rest of statute redundant
o formal salaried employee. Court rejects as too narrow
o **common law agency principles  uses this test. Can consider following factors to determine
if party is an employee: hiring partner’s right to control the manner and means by which the
product is accomplished – among the factors relevant to this inquiry are the skill required, the
source of the instrumentalities and tools, location of the work, the duration of relationship
between the parties, whether the hiring party has the right to assign additional projects to the
hired party, the extent of the hired party’s discretion over when and how long to work; the
method of payment, the hired party’s role in hiring and paying assistants, whether the work is
part of the regular business of the hiring party, whether the hiring party is in business, the
provision of employee benefits, and the tax treatment of the hired party…
 no one of these factors is determinative

29
 don’t need to weigh all of these
 this is technically outside copyright law, looking at state common law

30
FORMALITIES
(skim 651-665; read 666-686)
*what to do to obtain a copyright/protect
*this is the part of copyright law where there are right answers

History (how to obtain copyright)


 historically, had to publish the work, provide notice, register the work with copyright office, deposit copies
– and failure to conform w/these formalities at time of publication meant the work fell into public domain
 publication was key  that was the moment that the work became relevant (when work wasn’t yet
published, federal copyright law had no interest in the work at all)
 1909: Congress made formalities easier, still had to register the work. Publication still required. Had to
renew the work during the final year of first term. Failure to comply with formalities meant the work went
into public domain.
 1976: Congress envisioning itself joining the Berne Convention. Congress don’t think formalities as
important to copyright protection, begins to soften its rules.
o big change  less focus on publication-centered regime and more focus on fixation
o now, a work is protected the moment it is fixed
o notice still required but can cure if you forgot
o registration can happen at any time
o note: operative date for 1976 Act is January 1, 1978 (anything earlier is under 1909 Act)
 1989: pass Berne Convention Implementation Act (based in natural rights principles and hostile to the
notion that formalities are essential to copyright ownership)
o further softens formalities requirements
o notice no longer necessary
o registration not necessary, but to file lawsuit – is necessary
o if you register prior to the infringement, owner can claim statutory damages. Then, don’t need to
prove actual damages!
 default rule change is huge  before, you had to opt in to get copyright; now, you have to opt out (take
affirmative steps to say “no copyright in my work”

Purpose of Formalities
 provide notice
 tell ppl how long a copyright will last
 gives information about who you need to contact for licensing
 altho formalities low/cheap/didn’t require substantial efforts – still a meaningful hurdle in obtaining
copyright

The four regimes for copyright protection


1) 1909-1964 (1909 Act)
a. Notice,
b. Deposit
c. Registration in order to sue
d. Renewal
e. Failure to do upon publication or within 3 months meant work fell into public domain
2) 1964-1978
a. Renewal became automatic (don’t have to worry about filing renewal slip in the final year)
3) 1978-1989 (Berne Convention)
a. Notice is required, but curable
b. Deposit is required, but you can pay a penalty
31
c. Registration is still required to sue
4) March 1, 1989 –present
a. Registration and deposit required to sue
b. Notice gets you better remedies
c. But everything else gone, and not having notice, registration and deposit will not prevent you from
getting a copyright

Limited Publication
Estate of MLK v. CBS (per 1909 Act) (656)
F: MLK gave “I have a dream speech” and MLK go copyright protection a month after the speech under 1909
Copyright Act. 20 years later, CBS produced a documentary which contained footage of the speech (about 60%
of its total conent) and MLK’s estate sued.

I: Did the public delivery of the speech constitute general publication so as to place it in the public domain, or
was King’s attempt to obtain protection in 1963 succesful?

Note: Court decides only general publication will divest a common law copyright

H: This was limited publication due to distribution to the news media. Speech wasn’t publication. What
mattered for publication was whether the work was made freely available to the public via copies.
Dissemination to journalisms not wide enough dissemination. And merely reading the speech, even to millions
of people – since they didn’t have copies – is not publication

Oscar statute (Prof: just need to know this is an issue – don’t have to know it all)
-not published bc only given to some ppl (not freely available)
-general vs. limited publication
-we have to think about whether formalities were complied with when it was published –> 1) when might this
work have been published? 2) when would formalities have been complied with?
-due to challenges of formalities, in the 1990s, Congress allowed the owners of certain foreign works whose
works had fallen in to public domain due to lack of complying w/formalities to repossess the copyrights of those
works (including Tolkien works)
-didn’t do this for American works
-constitutional per Golan v. Holder (p683)

Duration
(666)

**remember, two different regimes of protection: (1) 1909 Act; (2) 1976 Act
*Plus, in 1998, the Copyright Term Extension Act, which amends the 1976 Act

Duration Charts

Works created after January 1, 1978: (effect date of 1976 Act)


*per 1976 Act, as modified by the CTEA
 Single author  Life of author + 70 years
 Anonymous authors  120 years from year of creation or 95 years from year of publication (whichever
expires first)

32
 Works made for hire  120 years from year creation or 95 years from year of publication (whichever
expires first)
 Joint authors  life of last surviving author + 70 years
 *Expires on last day of the year of the term (December 31) irrespective of when written or when author
died

Works created (but NOT published) before January 1, 1978:


 As of 1/1/1978, common law copyright abolished
 Duration is the same as for works post-1/1/1978, AND
o No copyrights can expire until 12/31/2002
o If published before 1/1/2003, at least until 2047
 ^Providing incentive to people to publish unpublished works by granting addt’l 45 years
of protection if the work was published before between 1978 and 12/31/2002

Works created and PUBLISHED before January 1, 1978


 If work was in public domain before 1/1/1978, it stays that way (except restored foreign works)
 If not in public domain, protection can last up to 95 years
o 28+28+19+20=95
 Two 28-year terms under 1909 Act
 19-year addition (to renewal period) from the 1976 Act
 CTEA in 1998 adds another 20 years to renewal period
 Works in their first 28-Year-Term on 1/1/1978
o If published prior to 1964 (through end of 1977):
 Renewal certificates had to be filed
 If filed, received renewal terms of 47 years (28 +19) plus 20 more in 1998, leading to 95
years of protection
 If didn’t renew…public domain after initial term
o If published in or after 1964:
 Renewal is automatic, though filing has benefits (e.g., control over derivative works)
(4) 95 years of protection

Works created before 1923


All in public domain

Notes and Case


Notes on Published Pre-1/1/1978 Works
 Publication w/proper notice marked beginning of 28-year term
o at end of this term, could file for renewal for another 28-year term
o date of author’s date irrelevant
 1976 Act extended protection for these works, adding add’tl 19 years to the renewal term
 In 1998, Congress extended the duration of protection by 20 years
o Longest possible duration for works prior to 1/1/1978 is 95 years
 In 1992, Congress eliminated the renewal requirement (from the 1909 Act), which had said that if copyright
holders didn’t renew, their protection would cease at the end of the initial term.
o Result: all works published in or after 1964 (until 1977) were automatically renewed

33
 But works published before 1964 still required filling of renewal certifactions (or else went to
public domain). So if publichsed between 1923-1963, need to check for renewal!

Notes on UNpublished Pre-1/1/1978 Works


 1909 Act only protected published works, but 1976 Act protets published and unpublished as long as fixed
 The 1976 Act granted protection for all previously unpublished works, granting life of author + 70 years
o AND no copyright can expire before 12/31/2002
 Also, provided incentives to ppl to publish unpublished works by granting additional Y45 ears of protection
if the work was published before December 31, 2002

Notes on 1976 Act


 Extended copyright renewal of 1976 Act by 19 years, total OF 74 years
 Transitioned from publication to fixation regime; works protected once fixed
 Enacted provisions for unpublished works
o Congress wants to give incentives—if you publish between 1978–2002, you get life of author + 70
o If however work was unpublished and remained unpublished term would be life of author + 70 but
would no expire before 2002
 Nothing will fall into public domain voluntarily until January 1, 2019

Copyright Term Extension Act


Eldred v. Ashcroft (672)
F: Congress enacted the Copyright Term Extension Act (CTEA), extending the duration of all future and
existing copyrights to the lifetime of the author plus 70 years (basically an additional 20 years). Plaintiffs
challenged the law, arguing that the extension of the terms of existing copyrights violated the Copyright Clause
in Constitution, because Copyright Clause only allows Congress to grant copyrights for “limited times.”

Plaintiff argument
1) Structural/textual – Act violates “for limited times” language
a. Concern about doctrines that don’t really seem to have an end (remember Lopez and Morrison –
struck under Commerce Clause – there has to be a limit)
b. Court doesn’t acknowledge this argument in their opinion
2) “Promote progress of science” violation
a. Question as to whether this clause is a limitation or constraint of what Congress can do
b. Economically, if we give ppl incentives, they’ll write more works; problem is that all of the
works that already exist get no new incentives – provide no new value to those works/ppl
c. Other things Congress may have considered when passing the Act  encouraging distribution
(modernizing old works that have fallen into public domain), people living longer…
3) Violates First Amendment
a. Court says copyright law itself has protective First Amendment doctrines (like idea/expression
dichotomy; fair use)
b. Court doesn’t seem to fully understand P’s argument here – says that it doesn’t think P’s speech
interest is so great when just saying other ppl’s words

H: Statute upheld. Applying rational basis review, Congress’ reasons are reasonable (opinion is mostly just
about deferring to Congress).

Dissent (Stevens): this is an abdication of the court’s role in judging the legitimacy of copyright regulation.
Court is basically saying that as long as Congress has a reason, we’re just going to accept it – no meaningful
oversight.

34
Dissent (Breyer): Act has no real benefit (small incentive) but huge cost. Most works are completely valueless
by the end of their term, so adding 20 years won’t do much. But the burden after the fact is huge – will cost way
more for consumers bc certain works that are already valuable will become so much more valuable, and
consumers will have to pay way more for them.

35
INFRINGEMENT (part 3)
(247-268; 270-304)
*now that you have a copyright, who can you sue?

General
 a plaintiff in an infringement case has the initial burden of producing sufficient evidence demonstrating (1)
ownership of the copyright; (2) violation of one of the exclusive rights reserved to the copyright owner

§501: Infringement of copyright:


“Anyone who violates any of the exclusive rights of the copyright owner as provided by sections 106 through
122…is an infringer…”

§106: Exclusive rights in copyrighted works


Subject to sections 107 through 122, the owner of copyright under this title has the exclusive rights to do and to
authorize any of the following:
(1) to reproduce the copyrighted work in copies or phonorecords;
(2) to prepare derivative works based upon the copyrighted work;
(3) to distribute copies or phonorecords of the copyrighted work to the public by sale or other transfer of
ownership, or by rental, lease, or lending;
(4) in the case of literary, musical, dramatic, and choreographic works, pantomimes and motion pictures and
other audiovisual works, to perform the copyrighted work publicly;
(5) in the case of literary, musical, dramatic, and choreographic works, pantomimes, and pictorial, graphic,
or sculptural works, including images of a motion picture or other audiovisual work, to display the
copyrighted work publicly; and
(6) in the case of sound recordings, to perform the copyrighted work publicly by means of a digital audio
transmission.”
^sets forth the exclusive rights of a copyright owner

Reproductive Right
(247)

General
 relevant part of statute: §106(1)
 this right, the exclusive right to reproduce the copyrighted work in copies/phonorecords has been
understood as the core right of the copyright owner
 copyright infringement is a strict liability tort (if you do it, you’re responsible)
o innocent infringement is a very narrow escape
o tho de minimus doctrine may be a defense: if what the defendant took was de minimus – too little to
justify a finding of copyright infringement – courts will not find infringement
 easy cases are exact copies/pure piracy. The challenging questions arise when D has created something kind
of like the P’s work, or incorporated part(s) of P’s work into D’s work
o in cases of nonexact copying, the question is whether defendant has engaged in actionable copying
(in violation of 106(1)) by taking too much of what is protected by copyright in plaintiff’s work
 scope: important aspect in infringement cases because you’re deciding the scope of P’s copyright in order to
determine if P can prevent someone else from doing something
 note: besides piracy, exact copying generally copies up in “privileged” activities, where the infringement
liability will hinge on the rules that govern the various limitations and exceptions to copyright. These are set
forth in §107-122 (see more on page 249)

36
o ex: fair use, copying for non-profit libraries, first sale, reproductions for disabled blind, broadcast
“ephemeral” use

Reproductive Right Attack Outline


**Plaintiff must establish all of the following to make a prima facie case**
(1) Must have valid copyright
(2) Prove there was a violation of the reproductive right; two concepts here (*don’t mix up – courts often do!)
a. Copying-in-fact: did defendant copy in fact or was there independent creation? (binary question)
1. Access: a “reasonable” opportunity to view or to copy plaintiff’s work (Bolton)
i. Can prove with direct evidence, that defendant had access (Steinberg)
ii. Can prove with indirect evidence, wide dissemination & subconscious copying
(Bolton)
iii. Can also prove through chain of events between plaintiff’s work and defendant’s
access to that work (such as through dealings with publisher or record company)
iv. Lack of access alone not enough to prove independently created, but is strong
evidence of not copying (e.g., poem locked in monastery, no one has ever seen it)
2. Striking similarity
i. Consider if the songs sound similar bc neither are unique, rather are just scenes a faire
– typical of the genre (Gibb)
ii. Even if no positive evidence of access, will infer access where works are so similar to
each other (and not to anything in public domain) that it’s likely the creator of second
work copied the first – and this was not an accidental independent creation. (Ty)
a. Sometimes, the similarity will be due to public domain
b. Illicit copying (substantial similarity): given that defendant copied, how similar must the works be
in order to find a violation plaintiff’s right?
1. Defendant copied too much/inappropriately, or just the “wrong” kind of copying
i. not all copying is “bad”
2. If defendant only takes stuff that isn’t copyrightable, no infringement (Nichols)
3. Qualitative and quantitative analysis:
i. Qualitative: can be literal or fragmented literal similarity (Harry Potter) – direct
quotations and close paraphrasing of vivid passages
ii. Quantitative: the facts/quotes taken were so essential to Seinfeld (Castle Rock)
4. Spectrum/scale of substantial similarity; end of spectrum is de minimus copying/no
infringement (What Women Want)
5. 9th Circuit: bifurcated approach
i. Extrinsic: based on formal criteria – compare specific expressive elements like plot,
theme, mood, dialogue (Cavalier)
a. Remove the noncopyrightable stuff (Carey)
ii. Intrinsic: response of an ordinary reasonable person
6. 2nd Circuit: holistic approach
i. Steinberg: ordinary observer
ii. Boisson (public domain aspects): more discerning observer
iii. Mannion: even if photo has some uncopyrightable elements, look at the total look &
feel of the photo
7. Think: should the original author of this work get to control this similar copying?

Note: You can have copying in fact but NOT illicit copying (i.e., only copied de minimus, or public domain or
scenes a fair)

37
Qualitative analysis of reproductive right
Harry Potter Lexicon, Twin Peaks --this is a class of cases that come up together. Literal, precise copying of a
section. And that's what this is. Approach is different than in say, Nichols case, where was nothing really similar
about these in terms of dialogue etc., yet they may have felt the same in terms of mood, plot, etc. these are all
ways of potentially violating plaintiff's reproductive rights. But due to the different kinds of copying, must
figure out which test to apply. Depends on what on copying we're facing! Ex: the fragmented literal
similarity (Harry Potter, Seinfeld), or more general, genre-based copying (then apply a different test)

De Minimus Copying
De minimus use not infringement
What Women Want
Gottlieb Development v. Paramount (253)
F: What Women Want movie had scene where a pinball machine was seen in the background. Gottlieb, the
designer of the art/logo on the pinball machine, claims the movie infringed on his copyright by reproducing his
art/logo in their movie. Defendant movie argues that it’s use was de minimus.

H: No infringement after substantial similarity analysis – altho copying - use of art/logo was trivial/de minimus.
(1) Quantitative (duration & amount – how much of plaintiff’s work has defendant used?)
a. Courts will consider the length of time the copyrighted work is observable, as well as factors such as
focus, lighting, camera angles and prominence
b. Amount of time relative to the work as a whole
c. “No plagiarist can excuse the wrong by showing how much of his work he did no pirate” (Learned
Hand) – just bc you add your own stuff doesn’t mean you still didn’t copy plaintiff’s work
(2) Qualitative (how was it used – observable, prominent?)
a. Altho art often visible in background of shot, it is often out of focus – can’t see the design clearly.
i. Always in background, never close ups
ii. Almost always partially obscured by actor or furniture
iii. Only fully visible for a few seconds during the entire scene
b. An average observer would not recognize the designs as anything other than generic designs in a
pinball machine: the unique expressive element of the design is not discernable
c. Pinball machine and its design is unrelated to the scene/movie’s message, plot, etc. no character ever
refers to it.
d. Short amount of time – 3 and a half minute scene
i. Appears in the scene sporadically, for no more than a few seconds at a time
e. Didn’t impact the plaintiff in any meaningful way (someone isn’t going to see this movie and then
think, OK I’ve basically played/seen the pinball game too).

Policy behind de minimus: don’t want to impose liability on everyone

Copying in Fact
Lots of access  subconscious copying
Love is a Wonderful Thing
Three Boys Music v. Michael Bolton (259)
F: Isley Brothers claiming Bolton copied their song, jury held Bolton copied and he appealed.

38
H: Affirmed that Bolton copied. Altho not much evidence of similarity, LOTS of circumstantial evidence of
access (subconscious copying by Bolton). As kid, Bolton was a fan and listened to Isley Bros. music and sang
their songs. The Isley Bros. music used to be very prevalent/on radio where Bolton grew up. A tape shows that
Bolton wondered if he was copying a song by Marvin Gaye (thought he was subconsciously copying another
song). Finally, Bolton was unsuccessful in providing independent access -- couldn’t overcome presumption
once plaintiff established reasonable access and similarity – with evidence of work tape and history of
songwriting.

My Sweet Lord case


Court held that George Harrison subconsciously copied Chiffons’ song after Harrison admitted to hearing song
several years ago when it was a hit. In that case, the court also found that the two songs were very similar.

No proof access + sound similar (but rock songs have “typical” way of sounding) = no infringement
Bee Gees
Selle v. Gibb (262)
F: Selle (small time musician) claims the Bee Gees infringed by copying his (not-widely distributed) song “Let
it End” with their song “How Deep Is Your Love”. At trial, Bee Gees introduced evidence of their creative
process, including testimony and a tape documenting the creation of their song. Selle’s expert music witness
testified that the note compositions in the two songs were so similar, that there must have been copying. Jury
found for Selle, but judge granted Bee Gees’ motion for new trial, finding that Selle failed to prove the Bee
Gees had access to his song.

H: Affirmed in favor of Bee Gees. Despite the similarities (chord structure, notation), there’s no proof of
access. Plus, they could just sound similar bc both from rock genre – but still be created independently.

Access
 Selle is a nobody, no one has heard his songs
 Only potential access situation is when Selle sent copies of his demos to record labels – he’s alleging
someone at record label heard it and copied

Striking similarity
 Melodies do sound pretty similar
 But we can’t just look at the number of identical notes; must also consider the uniqueness of the sections
which are asserted to be similar.
o The similarities should appear in a sufficiently unique or complex context as to make it unlikely that
defendant’s work is a matter of independent creation
 Note: this is PARTICULARLY important in the popular music field, where songs are
relatively short and tend to build on or repeat a basic theme
 Neither songs are unique – there’s no evidence or testimony that shows the relative complexity or
uniqueness of the compositions
 Rather, these songs are both “typical” scenes a faire
o There are many fundamental features/styles of particular genres of music (here, rock music)
o Similarity isn’t all the matters  fact that neither are “unique” also matters.
o Prof: they’re all just copying Chuck Berry!

No proof of access but VERY similar  copying in fact


Beanie Babies
Ty v. GMA Accessories (264)
F: Ty made a Beanie Baby pig and GMA made similar stuffed pig. Ty sued for infringement.

39
H: Although there was no positive evidence of access, the similarities between the pigs are so suspicious that
the court is basically assuming access. We will infer access where the works are so similar to each other (and
not to anything in the public domain) that it is likely that the creator of the second work copied the first, and that
this was not an accidental independent creation. This inference can be rebutted by disproving access or
otherwise showing independent creation (which GMA didn’t do here).

Public domain discussion


 GMA’s pig is strikingly similar to Ty’s, but not similar to anything in the public domain, such as a real pig
or public domain stuff animal or character. So the similarities clearly don’t come from fact that they’re both
just emulating real pigs
 In some situations, a similarity can be striking without being suspicious (ex: two ppl photographing Niagara
Falls from the same place on the same day, time)) – no inference of access to anything but the public
domain so no inference of copying from a copyrighted work

Does it matter, for purposes of copying in fact, that the thing(s) that are similar between the P and D's
work is not a copyrightable thing?
 The thing that the D copied is not an original, protectable aspect of the work, but nonetheless gives rise to
the inference that they there was copying
o **This is not the case when we talk about illicit copying/substantial similarity - those cases only
concern aspects of the work that are owned

Illicit Copying
General
 Question here is whether the defendant has engaged in actionable copying by taking too much of what is
protected by copying in the plaintiff’s work
 The substantial similarity test is difficult to apply, partially because the similarities are also relevant to the
threshold question of copying in fact
o Mostly tho, because the judgments involved are highly contextual
 This test is about the degree of resemblance between the works that supports a finding of infringement
 Note the aspects of plaintiff’s work which are not protectable (public domain, ideas, scenes a faire) and how
in different cases, the courts guard against the risk of finding liability when defendant only took plaintiff’s
unprotected materials

No infringement where only thing in common was high-level plot


Cohens and Kelleys
Nichols v. Universal Pictures (271) (2nd Circuit)
F: Nichols wrote the play “Abie’s Irish Rose” with the basic plot of a Jewish boy marrying a Irish Catholic girl,
their fathers disagreeing with each other, religious differences, and then the couple eventually having a baby.
Universal made movie “The Cohens and the Kelleys” about an Irish boy marrying Jewish girl and their families
not getting along (less on account of their religions, more about the general animosity of the Jewish father).
Nichols sued Universal for copyright infringement.

H: No infringement. What Universal “took” from Nichols is not copyrightable. High-level plot outline can’t be
copyrighted. The works are similar in only very high-level ways: Irish and Jewish families, grandchildren,
reconciliation. Aside from themes, the works are quite different – very different stories and characters. These
cases are similar on an abstract level of general ideas, not similar in their particularized expression.

40
Think: can’t copyright a “star-cross lovers” plot, but a more detailed plot may be protectable. Can’t copyright
the “grumpy father” character, but can likely copyright a specific character – Homer Simpson

Note: moving more toward judgments of consumers, less dependent on what experts think

Ordinary Lay Listener Test


Arnstein v. Porter (275) (2nd Circuit)
Crazy songwriter accused Cole Porter of infringing on his compositions. The lower court granted defendant
(Porter)’s motion for summary judgment and this court affirms. The standard used, did Porter take from
Arnstein’s work “so much of what is pleasing to the ears of lay listeners, who comprise the audience for whom
such popular music is composed, that Porter wrongfully appropriated something from belongs to Arnstein? This
test for similarity in infringement cases asks if a person would hear similarities in the things they value. Don’t
need experts to dissect the work measure by measure. Expert testimony may be relevant at the copying in fact
stage, but not the illicit copying stage. At trial, plaintiff may play (or cause to be played), the pieces in such
manner that they may seem to a jury to be inexcusably alike.

Second Circuit Approach


***Second circuit approach: (holistic) Ordinary Observer/More Discerning Observer***

Ordinary observer test


Moscow Poster
Steinberg v. Columbia Pictures (277)
F: Saul Steinberg claimed that the poster for the movie “Moscow on the Hudson” infringed on his copyright for
his New Yorker magazine cover. Columbia Pictures admitted they used the magazine cover as a reference but
argued that they didn’t copy so much/the kind of stuff which constitutes infringement.

Standard: whether an average lay observer would recognize the alleged copy as having been appropriated from
the copyrighted work

Columbia argument
 Amount of similarities: lots of things original in its work – street names, the actors images in front
 Scenes a faire: the two look similar bc they’re both of NYC; when you draw NYC, there are certain things
you draw bc that’s what NYC looks like  buildings, streets, lamps, rivers, etc. (Holmes: anyone can try
their hand at the original)

H: Works are substantially similar – infringement. Plaintiff wins on summary judgment (unusual).
 Amount of similarities: can still be infringement even if defendant work only involves a small portion of
work plaintiff’s work; the addition of original features doesn’t exculpate infringers (“no plagiarist can
excuse the wrong by showing how much of his work he did not pirate” – Learned Hand)
 Scenes a faire: The problem isn’t Columbia’s mere use of the NYC elements in its poster, but rather that
Columbia copied Steinberg’s expression of those elements in a street scene
o the idea – map of the world from an egocentrically myopic perspective isn’t copyrightable
o but the expression of it is, and here the styles are very similar  whimsical style of drawing, vantage
point, same structures/shapes, childlike spiky block print, shadows, squiggly lines, location of things

Note: court discusses fact that proof of access allows for somewhat less proof w/regard to similarity, but these
are two different tests – access is about copying in fact and the question here is illicit copying (two separate
but necessary elements of the reproductive right)

41
More discerning ordinary observer test (bc public domain)
Quilts
Boisson v. Banian (283)
F: Boisson is quiltmaker who created two quilts incorporating the alphabet. Banian made three very similar-
looking quilts, Boisson sued for copyright infringement.

Standard: “more discerning” ordinary observer test because the plaintiff’s work taken is from the public domain
(alphabet). This test is a more refined analysis, dissecting more carefully…but also don’t want to overly dissect
because then everything is in the public domain. (Consider: if you parse a novel too closely, you’re left with
words, then letters..and letters are public domain). The court should not dissect the works into separate
components and only compare the copyrightable element, but compare the “total concept and feel” of the
contested works.

H: One of the quilts is sufficiently similar. While defendant did some things differently (a different border and
different icons), a lot of the quilt is the same as plaintiff’s: some letters have exact same color letter and
background, some of the same icons, same letter shapes. Remember that the plaintiff can only protect the
aspects of the work which are original to him (so obviously not the public domain alphabet) – the color of the
letters/backgrounds, symbols, general colors.

Even tho some uncopyrightable elements, look at total look & feel of photo
Kevin Garnett photo
Mannion v. Coors Brewing (286)
See earlier facts

Defendant argues: it just copied the idea of a chilled out black guy wearing a white t-shirt and bling

Standard: looks like a total look and feel test (looking at the combination of all the aspects, protectable or not)

H: Substantial similarity, defendant copied Mannion’s expression.

Substantial Similarity
 the relevant comparison is between the protectible elements in the two photos (but these elements
shouldn’t be viewed in isolation)
o fact that Garnett photo includes certain uncopyrightable elements (cloudy sky, white t-shirt) does
not affect the nature of the comparison, bc the relevant question is whether the aesthetic appeal of
the two images is the same
 the public domain aspects aren’t copyrightable themselves of course but their existence and
arrangement still contributes to the photos originality
 the Garnett photo is protectible to the extent of its originality in the rendition and creation of the subject
o similar composition, angle, lighting, background, subjects wearing similar clothing and similar
jewelry arranged in a similar way

Court acknowledges that in visual images, the idea and expression may be the same thing.

Idea/Expression discussion
 rejects defendant’s argument that Mannion is trying to protect an idea – defendant clearly didn’t just copy
the idea, as the idea doesn’t account for all of the similarities between the two works – angle, pose,
background, composition and lighting

42
o one can imagine a number of depictions of a black man wearing a white t-shirt and bling that look
nothing like either of these two photos
 in general, the idea/expression distinction makes sense in the literary context, but the distinction breaks
down in the visual arts
o for one thing, it’s usually impossible to speak of the particular “idea” captured or conveyed in a
visual arts work bc every observer will have a different interpretation
o also, it’s not clear if there’s any real distinction between the idea and its expression – an artist’s idea
is to depict a particular subject in a particular way
o finally, the question of when similarities between two photos become sufficiently general that there
is no infringement even tho actual copying has occurred – is the same question as the substantial
similarity test for infringement (just phrased the opposite way)

Discussion of Kaplan v. Stock Market Photo


Defendant relies on this case – two very similar photos of a businessman’s shoes and lower legs, taken from the
top of a tall building, looking down on a street below. The court held that the photos were not substantiality
similar bc all of the similarities flowed from the unprotected idea, rather than from the expression of that idea

Ninth Circuit Approach


***Ninth circuit approach: Two step approach  extrinsic and extrinsic ***

Introduction of extrinsic/intrinsic test & fact that audience matters


Hamburger head infringement
Sid & Marty Krofft v. McDonalds (292)
F: Puppeteers/TV producers Sid and Marty Krofft alleged that the copyright in their TV show H.R. Pufnstuf
was infringed by McDonald’s “McDonaldland” ads.

H: Show too similar (total concept and feel) – infringement.


 Creation of two-part test:
o Extrinsic test: similarity of ideas based on specific criteria which can be listed and analyzed
 type of artwork involved, materials used, subject matter, setting for the subject
 analysis of specific formal criteria, as opposed to holistic, total concept/feel test
o Intrinsic test: substantial similarity test – response of an ordinary reasonable person.
 It is intrinsic bc it does not depend on the type of external criteria and analysis which marks
the extrinsic test
 And bc this is an intrinsic test, analytic dissection and expert testimony are not appropriate
 Audience: who counts as the audience of the work and how this may matter
o Courts do seem to care about who the consumer base is – can use this to argue similarity (or not) by
discussing who the consumers are
o Difference between adult and child consumers. These characters are for children and they’re less
discerning about these things. Here, see two big yellow guys, can’t see the difference (Prof: is this
really true?)

Separate analyses for literary and artwork


Talking moon kids books
Cavalier v. Random House (293)
F: Cavaliers wrote children’s book (Nicky Moonbeam) about an anthropomorphic moon and tried to sell it to
Random House, was unsuccessful. Later, Random House published books (Good Night, Ernie; Good Night,

43
Elmo) with an anthropomorphic moon and similar themes of a moon-character taking a child through a journey
in the night sky. Cavaliers sued for copyright infringement.

Two-part test
 Extrinsic (objective comparison of specific expressive elements like plot, theme, dialogue, mood, setting,
pace, characters and sequence of events)
o Must filter out the non-protectable elements
o These are measurable, yes-or-no, quantifiable characteristics of the work
o Can use expert testimony
o Looked at this level here (SJ motion)
 Intrinsic (subjective comparison focusing on whether the ordinary, reasonable audience would find the
works substantially similar in the “total concept and feel of the works”)
o This analysis would occur at jury level

Two separate inquiries


 (1) compare books as literary works taken as whole
 (2) compare the individual artworks

Comparison of literary works as whole (on SJ motion, only extrinsic test matters)
 After filtering out basic plot ideas, night sky setting, talking moon (scenes a faire) – look at the formal
feature – these narratives are very different
o Nicky: elaborate story lines, way more words, sophisticated language; Ernie/Elmo: little dialogue,
short, discrete scenes
o Altho stories have some events in common, are in different context/sequence
o Pace, dialogue, mood and themes different (Nightbeam is serious, instructional whereas Ernie/Elmo
is lighthearted and fun)  total concept and feel very different
o Characters different (except talking moon)

Comparison of individual art works (on SJ motion, only extrinsic test matters)
 Three artworks examined: (1) moon night light design; (2) illustration of stars relaxing on clouds; (3)
illustration of stars being polished
 Filter out the noncopyrightable stuff and examine the formal features
o Stars in the sky is obviously not copyrightable – is a fact/not original to plaintiff (it’s not like he’s
portraying them under the crust of the earth!)
o But the way the stars are depicted may be relevant – some are dirty and being cleaned; some are
wearing clothes – look at the outfits and how they’re being polished
 The way the plaintiff chose to anthropomorphize the stars
o There are some similarities in the shapes, color, subject-matter, style , arrangement and details

H: literary works not substantially similar enough to go to trial but artwork similarities are sufficient to survive
defendant’s summary judgment motion.

Removed ornamental notes bc performative in composition © case


Ornamental Notes
Swirsky v. Carey (298)
F: Swirsky wrote song, “One,” which was minor hit. Later, Mariah Carey wrote a similar-sounding song that
became a hit, called “Thank G-d I Found You.” Swirsky sued for copyright infringement, claiming that the

44
choruses of the songs were similar. At trial, Carey moved for summary judgement and prevailed. Swirsky
appealed.

Note: There are two types of music copyright: (1) composition – the underlying music and lyrics, usually fixed
in sheet music; (2) sound recording of the work. This case is about the composition.

H: Swirsky satisfied the extrinsic test, surviving Carey’s motion for summary judgment.

Opinion:
 In the court below, Swirsky relied on music expert – and lower courts didn’t like the expert’s “selective”
methodology. In his analysis, he looked only at structural notes, and removed “ornamental” notes (part of a
singer simply customizing the song – performative aspects not to be considered here) and scenes a faire
aspects bc they were not part of the composition.
 His methodology led him to state that although the two choruses were not exactly identical on paper, when
examined in the structural context of harmony, rhythm and meter, they were very similar.
 This court says his methods of excluding certain aspects was not only acceptable, but essential.
 Scenes a faire: Rejected lower court’s determination that parts of Swirsky’s song were unprotectable due to
scenes a faire. Just bc Swirsky’s song and For He’s a Jolly Good Fellow have similar pitch sequences
doesn’t mean Swirsky’s song in scenes a faire. These two songs are in entirely different musical genres.

Note: One of the challenges with compositional copyright is to determine what formal elements should count as
part of the composition and what elements should not. This case gives us some ideas of what’s protectable in
music compositions: music lyrics, notes, time signature, melody. Less often, but maybe: tempo, harmonies,
arrangement, rhythm, beat, instrumentation

Blurred Lines
Robin Thicke v. Marvin Gaye
 Gaye didn’t write music (results in challenge telling the difference between composition & performance)
o after he sung it, someone wrote down the music and then got ©
 courts wants jury to only compare the original parts of Gaye song to the original parts of Thicke song (but
that’s kind of impossible…)
o bc court only looking at the original elements
 so what should the jury hear and how?
o what was sent to the © office?
o as recorded?
o the melodies of each song on the piano?

Think: should rhythm be considered part of composition? (with classical, yes). What about more contemporary
music?

Note: basically no fair use doctrine in music

45
DERIVATIVE WORKS
(308-320; 324-334)

Derivative Work Right Attack Outline


1. Must transform, recast or adapt
a. Different purpose doesn’t mean derivative work (Harry Potter)
b. Just bc based on another work doesn’t mean derivative work (Harry Potter, Beanie Babies
guidebook)
2. Adaptations or Sequels
a. Twin Peaks: elaborate recounting, transforming material from teleplays into guidebook -- violation
b. MicroStar: incorporating aspects of store and characters from the game, violation
3. No copying
a. Transformation can be a violation
i. Mirage: Cut out P’s work and made a new work -- > changing from one form to another
ii. Lee: Same facts but NOT derivative work violation

§101: Definition of Derivative Works


Derivative work: work based upon one or more preexisting works, such as a translation, musical arrangement,
dramatization, fictionalization, motion picture version, sound recording, art reproduction, abridgment,
condensation, or any other form in which a work may be recast, transformed, or adapted. A work consisting
of editorial revisions, annotations, elaborations, or other modifications which, as a whole, represent an original
work of authorship, is a derivative work.

General
 relevant part of statute: §106(2)
 Per §103(a), derivative works are independently copyrightable (see copyright validity section)
 Here, the question is – what is the scope of the author’s exclusive right to prepare and authorize others to
prepare derivative works based upon the copyrighted work
 In an infringement case, usually, a plaintiff will allege that the defendant’s work infringes both her
reproduction and derivative work rights -- as these rights are closely allied
 Derivative works right is broad/powerful right protecting all adjacent markets (think: Star Wars movie 
legos)
 Question: what is the scope of the right; what kind of tests can be applied to determine if defendant has
violated the derivative works right? Has defendant changed the work enough so that it’s not merely
derivative, and rather, is non-infringing?
 Think: how far does derivative right go to cover downstream market?
 Qualitative/quantitative analysis
 Examples of infringing derivative works: translating a novel into Spanish; an opera based on Othello

Reproduction right violation; no derivative works violation


Harry Potter Lexicon
Warner Bros. v. RDR Books (309)
F: RDR planning on publishing Harry Potter Lexicon (encyclopedia of Harry Potter). Warner Bros. (has
exclusive rights from Rowling to create film series) sued, arguing that the Lexicon infringed on their Harry
Potter copyright – reproduction and derivative right.

H: Prima facie case of violation of reproduction right. But didn’t violate derivative works right.

46
Reproductive Right analysis (qualitative and quantitative)
 altho the works are different – i.e., no one would read the Lexicon instead of HP, the appropriate test is
whether the copying, qualitatively and quantitatively is sufficient to support the legal conclusion that
infringement occurred

Qualitative analysis
 each entry in the Lexicon contains fictional facts from Rowling’s expressions
 defendant argues that the order in which the fictional facts are arranged is different than the HP books, but
this alteration doesn’t mean no substantial similarity.
 But courts must also look at fragmented literal similarity (localized similarity in language) (Twin Peaks)
o here, the court examined the defendant’s considerable direct quotations and close paraphrasing of
vivid passages from HP. Often, Lexicon changed a few words from the original or rewrote the
original dialogue in the 3rd person – but the language is nonetheless substantially similar
o not as much as Twin Peaks case which was essentially a detailed recounting of the episodes, but still
sufficient

Quantitative analysis
 consider the amount of copying not only of direct quotations and close paraphrasing, but also of all other
protectable expression in the original works
 most of the Lexicon’s entries contain direct quotes or paraphrases, plot details or summaries of scenes--
thousands of fictional facts from the Harry Potter works.
 Lexicon draws 450 manuscript pages worth of material primarily from the 4,100 Harry Potter novels
 also reproduces substantial portions of Fantastic Beasts and Quidditch Through the Ages (even more than
the novels, as these companion books are much shorter and Lexicon reproduces a substantial portion of their
content across hundreds of entries)

Derivative works analysis


 work had an entirely different purpose, didn’t “transform, recast, adapt”
 just bc “based” on HP books isn’t enough to be deemed a derivative work (if that were the rule, then book
reviews and parodies would fall under that definition!)
 different than the derivative rights violation in Twin Peaks guidebook case where Twin Peaks plot
summaries gave such elaborate recounting of plot details that it was an abridgement of the original work,
simply transforming the HP material from one medium to another
o Lexicon isn’t an “elaborate recounting” and doesn’t follow same plot structure as the HP novels; it
also doesn’t merely transform the HP material from one medium to another
 Lexicon condenses, synthesizes and reorganizes preexisting materials – doesn’t recast the material in
another medium to retell the story of HP – it gives the copyrighted material another purpose  to give the
reader a ready understanding of individual elements in the elaborate world of HP

Discussion of Ty Inc v. Publications (Beanie Babies guidebook)


 collectors guide to Beanie Babies not a derivative work bc didn’t recast, transform or adapt Beanie Babies
 The text (descriptions/info about the Beanie Babies) uses the underlying work, but isn’t a derivative work.

Discussion of Twin Peaks (guide to the series)


 89 lines of dialogue copied from the Twin Peaks teleplays constituted substantial similarity
 “fragmented literal similarity”
 46 pages of a chapter in the guidebook was essentially a detailed recounting of the first 8 episodes of the
series – every intricate plot twist and element of character development, in the same sequence as in the
teleplays
47
 guidebook was derivative work – contained a substantial amount of material from the teleplays, transformed
from one medium to another

Discussion of Castle Rock v. Carol (Seinfeld SAT)


 this case was a reproductive rights case (not derivative)
 the items themselves are very different (trivia book obviously isn’t a substitute for the show), but used the
qualitative/quantitative approach and found that substantial similarity. This is a market only the plaintiff
should enter
o qualitative: facts taken were so essential to Seinfeld
o quantitative: trivia book copied 643 fragments from 84 episodes – lots of quotes

How could D have avoiding this (HP) litigation?


 entries that didn’t paraphrase/copy so much. Difficult to do tho…how to describe dementor in a different
way when the language itself is a part of the work

Note: Rowling had stated an intention to create an encyclopedia for the series. This isn’t really an important part
of the legal analysis, rather it’s just part of the court’s understanding of the nature of the market (note that court
treats different markets differently).

Derivative Works With No Copying


Violation after cutting out and repurposing tiles
Tile case #1– interpreting “transformation” in 101
Mirage Editions v. Albuquerque ART (324) (9th)
F: D bought copies of book with P’s work, cut the works out, glued them onto tiles and sold them. P sued and D
argues (1) its tiles are not derivative works; (2) first sale doctrine precludes a finding of infringement.

H: Violation of derivative works right due to transformation of P’s images onto D’s tiles. The work was in the
book, now it’s on tile – changing from one form to another form, making another version of plaintiff’s art work.
Rejects the first sale argument, the right to transfer applies only to the particular copy of the book defendant
purchased and nothing else.

No violation after cutting out and repurposing tiles


Tile case #2 – interpreting “transformation” in 101
Lee v. ART Company (326) (7th)
F: D bought P’s work and mounted it on ceramic tiles and resold. P sued.

H: No violation of derivative works right. No transformation – just depicting something exactly as it already
was. If framing a work doesn’t creative a derivative work, then neither should mounting art on a tile. The
defendant did not recast, adapt or transform plaintiff’s work. First sale doctrine doesn’t apply/cover the
derivative works right.

Fan-created levels were “sequels” -- derivative works


Duke Nukem game
Micro Star v. FormGen (329) (9th)
F: FormGen made video game Duke Nukem where players could design their own levels (MAP files) with
computer code, then post them on internet so others could download/play. These fan-created filed were stored
as data files in the main game program (didn’t contain any art or images from the games, rather, consisted of
code which told the engine –“put this ghost here,” etc.) MicroStar collected a bunch of these fan-created levels,
48
packaged them up and sold on CD. FormGen claims the levels were derivative works, and as the copyright
holder, only it can license derivative works.

Micro Star argument: relying on Galoob, argued (1) data in the level files not fixed in a concrete/permanent
form; (2) the level files don’t copy any of the game’s actual expression

H: Violation of derivative work.


 First of all, level data was fixed – on a CD.
 Altho the levels didn’t contain any of FormGen’s art or images, it was still a derivative work bc were like
maps of how to get that protected material. Reading a MAP file would basically be like reading what
happens in the game.
o Comparison to reading music composition notes – when read them, you hear the music in your head.
Here, if you read the code, it’s as tho you’re playing the game. The file is the blueprint.
 The MAP files incorporate aspects of the Duke story – protected expression.
 These levels are essentially “sequels” – continued versions of the game, incorporating aspects of the story
and characters from the game.
 This is different from Galoob, where the game genie allowed players to alter individual features of a game,
such as character’s strength or speed by “blocking the value for a single data byte” and replacing it w/a new
value. In Galoob¸ the change was temporary and when the game was over, the changed “version” was gone.

Hypos related to MicroStar


 Publish lyrics to song online, and write that it is to be sung to the tune of Beyoncé song “Love on Top”.
Infringe on derivative works right in the Beyoncé underlying composition?
 What about writing about what happens in a ballet in extreme detail?
 Clearplay software: cut out “naughty bits” of movies while you watched them, sued for violating derivative
rights. Congress changed law to allow this, but what if they hadn’t? They’re not actually editing/change the
file on the DVD, they’re just changing what you perceive. Tho isn’t perception basically the same thing as
altering a work?
 Cut up all the pages of a book and put them back together in a different order?
 Collages from pages of a magazine?

Characters
(334-343)

General
 This is a subsection of the reproduction/derivative works right
 Question: do characters in literary/pictorial works have independent copyright protection?
o To what extent can they be used/reused in subsequent works?
 Remember, characters are also not protectable if not distinctive enough (Nichols: scenes a faire)

Characters Attack Outline


1. Scenes a Faire: If character is very general/not distinct enough (“grumpy father” “drunk Irishman”), it’s not
protectable (Nichols)
2. Sufficiently Delineated: character must be sufficiently delineated in order to be independently
copyrightable
a. James Bond has consistent, widely identifiable trains (Honda)
3. Wizard of Oz case
a. Characters sufficietnyl distinctive to get protection

49
b. Only aspects of character in public domain are the images from the publicity materials, not the
characters themselves bc posters don’t reveal anything about the character’s signature traits,
mannerisms, speech, personality traits, etc.

“Story Being Told” Test


Maltese Falcon
Warner Bros. v. CBS
F: Author licensed the rights of his novel The Maltese Falcon to Warner Bros. to create a movie based on the
book. Later, author wants to write more stories about the lead character of the novel, Sam Spade. WB argues
that this violates the copyright they now own, the character Sam Spade.

H: Author prevails. Court develops/applies the “story being told” test  only the characters that amount to the
story being told are protectible when you own the copyright to the underlying work. When the character is not
the “story being told” (it’s just a story which features the character), the character is not protected by copyright.

Notes:
 Seems like Court also just sort of feels bad for author
 “Story being told” test is a VERY high bar to copyright a character

“Sufficiently Delineated” Test


James Bond Honda commercial
MGM v. Honda
F: Honda commercial with James-Bond-like character. MGM (owns copyright to all but one of the Bond
movies) sues Honda, arguing it violated their James Bind copyright.

H: MGM prevailed bc their copyrighted films established a copyright in the character of James Bond. Court
asked whether the character was sufficiently delineated, and because he was, he is independently
copyrightable. James Bond has certain consistent, widely identifiable traits to deem him sufficiently delineated:
British, well-dressed, cold-bloodedness, marksmanship, over sexuality, love of martinis “shaken, not stirred.”

Consider: Lots of actors have played James Bond, so who is he, really? And then which one will we compare to
the guy in the commercial? Or is it scenes a faire, some handsome guy in a tuxedo? And what if the character in
the commercial was black, or female? Think about the evolution/variations of the character in order to think
about the scope of the character (understanding who he is). Maybe the different favorite car, different bow tie
size, different actor in every movie makes the character less distinct. Or maybe this is expanding the character
and James Bond is all of these things.

Note: “sufficiently delineated” test is easier to meet than “story being told”

Sherlock Holmes
Someone arguing that because the character is still being written about, the character has never really been “set”
– is still evolving/being developed. Court didn’t buy this argument, but is interesting to think about.

Anything in publicity posters is fair game (public domain), but the actual characters and their distinctive
character aspects are not
Wizard of Oz Characters
Warner Bros v. X One X (335)
F: WB owns copyright to films Gone With Wind, Wizard of Oz, Tom & Jerry, but not on some particular
publicity material images (e.g., movie posters, lobby cards) bc didn’t comply with formalities – so they’re in the
public domain. AVELA has acquired restored versions of the movie posters and lobby cards and extracted the
50
images of the famous characters, and is using these images on shirts, lunch boxes, playing cards, action figures,
etc. In many cases, AVELA has modified the images, such as by adding a character’s signature phrase from the
movie to an image modeled on that character’s publicity photograph. In other cases, AVELA combined images
extracted from different items of publicity materials into a single product. WB argues AVELA infringed on the
valid film copyrights. AVELA argues that the entire characters are in the public domain due to the publicity
photos.

The Issues: If material related to certain characters is in the public domain, but later works (here, the films) are
covered by copyright and add new aspects to those characters, a work developed from the public domain
material infringes the film copyrights to the extent that it incorporates aspects of the characters developed solely
in those films. Court must determine:
(1) the apparent scope of the character copyrights in the films
(2) the scope of the material in the publicity materials, which correspondingly limits the scope of the film
copyrights (essentially, how much of the characters is in the public domain)
(3) the scope into which each of AVELA’s images fall
a. If an AVELA work falls solely within the scope of the material in the public domain, there is no
infringement of the film copyrights
b. If some portion of an AVELA work falls outside this scope, but within the film copyrights,
AVELA is liable for infringement

Discussion and holdings:


(1) Film characters are copyrightable. The scope of the film copyrights covers all visual depictions of the
characters, except for any aspects of the characters injected into the public domain by publicity materials
a. The characters exhibit consistent, widely identifiable traits – are sufficiently distinctive to merit
character protection.
b. Character copyright protection is limited to the increments of character expression in the films
that go beyond the character expressions in the (public domain) books on which they are based
i. This distinction has little practical effect, since the book’s description of the characters
anticipates very little of the expression of the character in film.
(2) The only aspects of the character in the public domain are the exact images in the publicity
materials.
a. The depictions of the characters in the poster are sufficiently different than the actual characters
in the movies
i. 2D vs. 3D, different colors, most importantly  the poster characters don’t do or say
anything – we don’t know anything about them! Don’t know Tin Man doesn’t have a
heart just by looking at the poster.
ii. The posters don’t reveal anything about the character’s signature traits, mannerisms,
distinctive speech, movement, demeanor and other personality traits that combine to
establish a copyrightable character
b. Characters’ visual appearances in the posters do not present the requisite consistency to establish
any “copyrightable elements” of the characters’ visual appearances.
i. No individuals were depicted with consistent, distinctive visual characteristics throughout
the various publicity materials – are wearing different outfits, in different colors, etc.
(5) If a character was depicted w/consistent, distinctive visual characteristics
throughout the publicity materials, these consistent visual characteristics would
define the copyrightable elements of that film character – injected into the public
domain by the publicity materials
(6) But bc this ^ is not the case, there are no visual aspects of the film character in the
public domain apart from the publicity material images themselves
(3) Three categories of AVELA items
51
a. No infringement: Products which reproduce an image (or just a small portion of an image) from
the posters on t-shirts, lunch boxes, playing cards, etc. Doesn’t display an “increment of
expression” of the film character beyond the pictures of actors in costume on publicity materials
b. Infringement: Products which juxtapose an image from the publicity materials with another
image extracted from elsewhere in the publicity materials, or with a printed phrase from the
books – creating new composite works.
i. Ex: t-shirt with image of Judy Garland as Dorothy printed with the phrase “There’s no
place like home” – this increment of expression added to the (public domain) image
evokes the character and constitutes infringement of the copyrighted character. It invokes
the copyrighted character – suggesting the actual, protected character – going too far.
c. Infringement: Products which extend an image extracted from the publicity materials into three
dimensions (action figures, figurines, busts) with juxtapositions of composite images with
images/words like those in the category above.
i. In addition to the problems from the above category, these 3D products use visual
information from the films, where the characters are observable from a multitude of
viewing angles

Movie/book copyright discussion


 Scope of copyright protection for characters in the films is limited to the increments of character expression
in the films that go beyond the character expression in the books on which they were based
 so only things that are original to the movie are protected
 ex: witch’s shade of green in movie. After reading about her “color” in the book, one can’t visualize exactly
what color it is. So the shade of green in the movie is protected.

Distribution Rights
(354-363)

General
 106(3); Distribution: “The owner of a copyright has the exclusive right to distribute copies or
phonorecords of the copyrighted work to the public by sale or other transfer of ownership, or by rental,
lease, or lending.”
 Statute imposes liability on people who circulate copies of works – even when they didn’t make the copies
themselves
 Remember, strict liability – can be liable even if didn’t know you were violating the right
o Ex: A copies B’s work and then Amazon distributes A’s copies. Amazon has violated B’s
distribution right

First Sale Doctrine


 This doctrine is an important limitation on distribution rights
 §109(a): “Notwithstanding the provisions of section 106(3), the owner of a particular copy or
phonorecord (sound recording) lawfully made under this title, or any person authorized by such owner, is
entitled, without the authority of the copyright owner, to sell or otherwise dispose of the possession of
that copy or phonorecord.”
 Justification/benefits:
o Consumers more likely to invest in works if they know they can resell
o Law prefers to protect peoples’ possessory interest in their property – common law doesn’t like
servitudes/restrictions on alienation. Best way for property to circulate is free market
 Software (music, book, programs): you don’t actually own a copy so you can’t sell “your copy.”
52
o You’re just licensing those rights
o This is why CDs are cheaper than buying all the iTunes songs. When you own a CD, you can sell the
CD – you “own” that music. But on iTunes, your interest in the copy of the song is much less
 How original copyright owners deal with this doctrine
o coming out with new editions
o raising initial prices
o affect the nature of the physical embodiment of the copy.
 Ex: our textbook is meant to fall apart quickly – original copyright holder controlling access
to the work

First sale doctrine – copyright holder can’t control downstream prices


Selling book
Bobbs-Merrill Co. v. Straus (354)
F: Bobbs-Merrill owns copyright to the novel “Castaway.” BM sues Straus (Macy’s) to restrain the sale of the
novel at a retail price of 89 cents per copy. BM had placed a notice in the books that retail price was $1 and if a
dealer sold it for less, this would be treated as an incident of copyright infringement. BM is arguing that they
have the exclusive right to sell the work and control the nature of sales of the work.

H: Straus may sell it for 89 cents. Copyright law do not create a right to impose a limitation at which the book
shall be sold at by future purchasers. The copyright holder has a limited right to sell the work – it’s limited by
the first sale doctrine. BM can only sell the novel once – can’t control downstream prices – its interest in selling
is now exhausted.

Note: So what can I do with my copyrighted book? I can’t reproduce it or create a derivative work out of it. But
I CAN resell it, rent it, throw it out, etc.

Selling “used” music


Capitol Records v. ReDigi (356)
F: ReDigi is marketplace for selling "used" music files for less than the cost of music on say, iTunes. Files a
person sells must have been purchased on iTunes or ReDigi and the software attempted to prevent users from
retaining a copy of the music that the user sells by prompting users to delete it (and suspending accounts of
those who don’t comply.) Capitol Records, which owns a number of the recordings sold on ReDigi’s website,
sues, alleging that ReDigi infringed on its reproduction and distribution rights. ReDigi argues (affirmative
defense) First Sale doctrine. Also tries arguing that music should be circulated like books – good for society.

Note: ReDigi trying to make this like selling a book – new buyer can read it and the seller cannot. But here, is
difficult to control the original owner’s ability to still listen to the music. And even if they had been successful
in doing this – still problematic for other reasons.

H: Capitol Records prevails. ReDigi violated the distribution and reproduction right.
 First Sale doctrine only apples to distribution rights and ReDigi was reproducing – they made new copies
for the new owner – violating the reproduction right
 Court is concerned (and thinks Congress was too) about the scope of competition that the original copyright
owner would face if it found for ReDigi here
o Rejects ReDigi’s “this is just like used books” argument -- book world is very different than the
digital world….
o When you transfer copies of real world objects, there are time, space and effort constraints (finding
someone to buy it, going somewhere to buy it, etc.) which make the used version an imperfect
substitute for the old.

53
o Think: used books aren’t necessarily competitive with the original copyright owner’s markets. Lots
of ppl prefer new books – no markings, highlighting, good quality. So the book market is preserved
for ppl who value new books bc the used book market is imperfect competition.
o Plus, used book market isn’t very efficient so even worse competition (pre-Amazon…had to go to
used book store and look for the particular book you want, may not have the right edition, etc.)
o Music world very different than book world:
o In ReDigi’s world, of course you’re doing to buy the “used” 75 cent version of a song on ReDigi as
opposed to the “new” copy on iTunes.
 So easy: it’s not like you have to wait around at the record store waiting for a used record to
show up. Just go on ReDigi and type in the name!
 Plus, you eventually throw old books out (falling apart) but music doesn’t degrade.

54
DISPLAY RIGHTS
(398-406)

General
 This is another right owned by copyright owners (in addition to reproductive right; derivative rights;
distribution rights)
 Putting copyrighted material on a publicly accessible website constitutes display
 We don’t see a lot on display rights, mostly because there is a big limitation (109(c)) on this right  owner
of copyright has the exclusive right to display copies of the work, but if you own a lawfully created copy of
the work – then you can lawfully display it
o If I lawfully buy a lawfully created “Wizard of Oz” poster – I can display it
o Have to own it, limitation doesn’t apply to renters
 §101 Definition of Display Right: right to show a copyrighted work either directly or by means of a film,
slide, television image, or any other device or process or, in the case of a motion picture or other audiovisual
work, to show individual images non-sequentially.
 §109(c) Limitation on Display Right: Notwithstanding the provisions of section 106(5), the owner of a
particular copy lawfully made under this title, or any person authorized by such owner, is entitled, without
the authority of the copyright owner, to display that copy publicly, either directly or by the projection of
no more than one image at a time, to viewers present at the place where the copy is located.
 §110: More Limitations: public interest expectations pertaining specifically to the public dissemination
right; various aspects of complication in the copyright statute -- lots of exceptions, esp. for libraries, ppl
with disabilities etc.

Porn thumbnails
Perfect 10 v. Amazon (Google) (398)
F: Perfect 10, which sells access to naked photos, is suing Google for (1) displaying thumbnails of its
copyrighted images; (2) providing links to the copy of the copyrighted images. Perfect 10 says people are
copying their copyrighted images and putting them on other, freely excessive sites. Then Google crawls these
sites and displays these copyrighted images. (Perfect suing Google and not these little infringement websites
probably bc those little sites have no money or there are jurisdictional issues).

Note: When you use Google Images and click on thumbnail, two things happen  (1) the thumbnail gets larger
(process called in-line linking); (2) it’s bordered by Google Content

H: Thumbnails are prima facie infringing on Perfect’s display right; full size images do not.

Thumbnails: Thumbnails constitute prima facie infringement of Perfect’s display right (maybe reproductive
right too). “Display” in digital world requires that you have a copy on your server – which Google does. Google
essentially takes the images off the offending websites, copies it and shrinks it, and then that image is shown
(residing on the Google server)

Full-size images: Full-size images (after you click the thumbnail) do not violate Perfect’s display right. When
you click the thumbnail image, Google takes its own HTML code and directs you to the offending website –
bringing that website forward onto your screen – not through a Google Server at all. So although it looks like
it’s coming from Google due to the frame, nothing is being stored on the Google server. The photo is only on
the offending/original server. Doesn’t matter that consumers may be confused (that’s a trademark concept) – all
that matters is that Google did not display the work. Google basically saying “The work is over there on that
site”.

55
MUSIC
(409-435)

SEE CHART

The Two Music Copyrights


 There are two copyrights in any piece of recorded music, with different rights
o (1) musical composition (the underlying composition) under 102(a) (subject matter of copyright)
 Fixed music composition exists in, and is reproduced in copies
 Includes notes, melody, lyrics, time signature, melody
 Less likely to be protected (but maybe)  arrangement, rhythm, beat, instrumentation, tempo
o (2) sound recording (the tangible phonorecord of the music/sound)
 Fixed sound recording exists in and is reproduced in phonorecords
 Includes genre, tempo style, instrumentation

The “Players”
 Music publishers are middleman between the songwriter/composer and other industry players,
including performing artists, record companies, collective rights organizations and other entities that might
license a songwriter’s compositions
o Generally, the songwriter/composer assign the copyright to the publishers and the publishers split
royalties earned on the work 50/50
 Publishers copy and distribute sheet music
 Assignments can be terminated after 35 years
 Collective Rights Organizations (ASCAP, BMI, SESAC) license the public performance rights from
music publishers
 Record companies are middleman role for performing artists. Record company works with the artist to
produce, distribute and promote the recorded songs.
o Generally, the artists assign all copyright interest in the sound recording to the recording company in
exchange for royalties or other compensation
 At this point, distributors of recorded music enter the music industry picture – a record company may
distribute CDs to stores and will make the sound recording available to digital distributors (iTunes), which
then offers them to consumers via “digital phonorecord delivery”

Musical Composition Copyright & Covers (Derivative Right)


 Copyright law recognizes a compulsory license to make mechanical reproductions (recordings) of
compositions, often called “covers” (codified in §115)
o If I want to record a cover, just need to purchase the compulsory mechanical license (and the original
copyright owner can’t say no)
 Today, music publishers license their rights to make mechanical reproductions to the Harry Fox Agency.
Then Harry Fox offers these up to the world, remitting payment back to the copyright owners.
o So if I want to re-record my own version of “Like a Rolling Stone”, I don’t need to go to Bob Dylan
and insist on the specific compulsory license and then pay him a certain amount. I just go to the
Harry Fox Agency and they tell me what it costs (the cost of the mechanical compulsory license)
and I pay it and then I can record my cover song.
 Requirements for compulsory license – covers:
o Initial requirements: To be subject to the compulsory license, the music work must have been
previously distributed to the public and embodied in a phonorecord created under the authority of the
copyright owner.
56
o Requirements for the recording artist “covering” the song: The recording artist may make a new
arrangement of the work to conform to his or her style, but may not change the “basic melody or
fundamental character of the work” per §115(a)(2)
 Can’t change the melody or add a new verse
 The new arrangement is expressly excluded from obtaining protection as a derivative work
unless the copyright owner consents
 If you want to do something else, like add verses or synch the song with a movie, video game
or TV commercial, then you need to negotiate a particular license with the copyright holder
(usually the music publisher) – the mechanical compulsory license isn’t enough
 They can say no
 Consider: is this a good thing? To have a thousand terrible recordings of terrible christmas song? And if it is
a good thing, why not have a compulsory license for other things, like plays? Maybe we’re concerned with
tarnishment – a crappy version of Hamilton will make people think the work is bad. Then again…it seems
this isn’t a problem with music…

Sound Recording Copyright


 This is a new (1972) and narrow right, much narrower than musical composition right
o Came about in order to stop record piracy
 Definition of sound recordings in §101
 This rights protects the elements of original authorship that inhere in a fixed recordings of sounds, whether
it is a recording of a music performance, a dramatic reading or a sequence of railroad whistles
 Protects against duplicating sound recordings – literal duplication by copying that record
o doesn’t extend to duplicating the exact same sounds—your band can duplicate the sounds (“sound-a-
likes” are OK)
 Note: sound recordings are fixed in phonorecords, not copies
 Consider: Who are the authors?
o Musicians (making some judgments about how to play the song), sound engineer, singer, producer
o If I go outside and record the sounds of the street on my iPhone, I’m the author of that original sound
recording (then we’ll have to ask if it’s actually original – Feist, Bannion)

Sampling
(419)

General
 In certain musical genres, “sampling” is a common practice
 Sampling involves digitally copying and remixing sounds from previously recorded albums
o Producers take small bits of songs, sometimes manipulate them, and put them into another song
 Not about mash-ups
 This practice implicates two copyrights (musical composition and sound recording)

Sampling from composition was de minimus, no violation


Beastie Boys – Three Note Sequence
Newton v. Diamond (419) (2005)
F: Newton composed the song “Choir”. Beastie Boys paid for/licensed the right to sample a sound recording
from “Choir” in their song “Pass the Mic,” but did not pay for/license the right to sample the composition.
Newton sued the Beastie Boys for their sampling (using a three-note sequence which involves flute playing and
throat-singing), alleging infringement based on his composition right.

Note: Unclear why they didn’t get the right to sample the composition – maybe they didn’t know they had to.
57
H: use was de minimus, not actionable

Discussion
 Is this even a chunk of copyrightable expression? Yes
o Court says it may be original, plausibly, there’s something special enough
 We know there’s copying in fact
 Illicit copying
o Copying was de minimus. There was so little/so trivial copying
 First, we filter out all the stuff that’s not an issue here (aspects of performance, bc they’ve
paid for the sound recording license)
 So what aspects are performative vs. compositional? See Carey, think about what a generic
rendering of this composition would be (that’s what we have to compare, not the
performative aspects). What part of the chunk of music comes from Newton’s clever ability
to play and what part comes from what he’s written?
o Would a reasonable juror recognize that this sample was appropriated from Newton?
 Court says no.
 Quantitative: just 3 notes, a couple seconds (have to consider the percentage of the original
work that Beastie Boys took from, and what percentage of Beastie Boys work the original
work represents)
 Qualitative: 3 note sequence isn’t the “heart” or “hook” of the song (the chorus probably is)

No de minimus rule for sound recordings


“100 Miles”
Bridgeport Music v. Dimension Films (425)
F: Claim is that D sampled P’s song “Get Off” in D’s song “100 Miles”. D had licensed the composition but not
the sound recording. D argues his use was de minimus.

H: D violated. Either get a license or don’t sample. There’s no de minimus rule for sound recordings. Court
isn’t going to go through the whole substantial similarity or de minimus inquiry because defendant hasn’t
disputed that is digitally sampled the recording.

Reasoning:
 there’s nothing in the statute about de minimus!
 De minimus came about as common law and has been adopted in all other copyrightable works, but this
court is saying Congress didn’t mean for it to apply to sound recordings.
 Congress crafted a very narrow right for sound recordings – a right protecting against duplication of sound
recordings. And that’s exactly what D did
o The fact that Congress created such a narrow right – we have to take it seriously
 Court also reminds us that sampling is never accidental so we’re less worried about innocent infringement
 Remember, they can always go back and re-record! There’s no violation when you record a sound-alike.

Note: In Madonna case, the 9th Circuit completely disagreed with Bridgeport and found that there is a de
minimus right/exception

Public Performance Right


Music Composition Copyright

58
 musical work copyright owners have a public performance right; radio stations, concert venues, nightclub
owners and other businesses that “play” music need a license (110), unless a special exemption applies.
o Collective rights organizations (CROs) license the public performance rights from the music
publishers, and then administer blanket licenses for millions of copyrighted works (main CROs are
ASCAP, BMI, SESAC)
 Basically, if you want to perform a musical composition publicly, you’ll have to deal with
one of these organizations to get licenses (not the artists themselves). These organizations
have contracted with the music publishers (who split royalties 50/50 with authors).
 These organizations sell the rights to publicly perform these musical compositions – sell
them in bulk (not just one particular song)

Sound Recording Copyright


 The public performance right for sound recordings is extremely limited
 The right is limited to digital audio transmissions, like Pandora or Spotify (they have to pay to play the
songs but radios don’t)
o In fact, it’s long been believed that playing these songs on radio is like advertising for the musical
composition

59
FAIR USE (part 4)
(563-600; 609-631)

General
 Affirmative defense to infringement; general defense/limit that applies to all copyright rights
 Doctrine is a shifting set of factors – lots of room for interpretation and argument depending on situation
o So look at facts in front of you and then decide which precedent/cases apply
 Fair use originally emerged in common law (Folsom), eventually codified in 107
o Statute enumerates factors for courts to consider in making their fair use assessments
 Think: how does fair use influence plaintiff’s work and what can others fairly/freely do without
authorization?
 Justification: sometimes, use of another author’s work is important – must be opportunities for using
another’s work. When is an author’s work so valuable they we won’t “let” original author control it?
o may need it to further develop and improve upon their own work (“progress”)
o we want ppl to be able to critique another’s work -- incentivize ppl to write better; let ppl judge
what’s worth watching/reading
o doctrine is like stand-in for getting consent from authors (consent implied in some circumstances)
 of course, sometimes the author wouldn’t really consent – like a really negative review with
excerpts from the author’s book…but we still want ppl to be able to do this

Evolution of the Doctrine


 Harper: focus on consent and reasonable expectation of copyright owners
 Acuff-Rose: work must be transformative – look at how the work criticizes/comments on the original work
 Cariou: work must add a new meaning or message (doesn’t need to comment/critique the original). Focus
on the secondary work’s new meaning/message, and whether there is a separate market (not just superseding
the original work)
o Consider harm to the market – economic incentives that cause authors to do stuff in the first place
o Consider: what about offending/harming the plaintiff in a way that causes the plaintiff to stop
creating? Say, the nature of defendant’s work is just so appalling and gross. Is this an economic
incentive the court should care about?
o Remember: economic doesn’t just mean financial. Can also be about motivation – ALL the
incentives that an author has to create. When one of these incentives is harmed, this affects our
analysis.
 Technology cases think about fair use differently – the nature of what plaintiff did and how what the
defendant is doing is different than what plaintiff did
o Sega: defendant using plaintiff’s work so that it can create its own thing
o Perfect 10: defendant not using plaintiff’s use for its entertainment value, but rather to direct
 Defendants can argue that it’s “important” for them to be able to use or copy plaintiff’s work
– not to compete – but because they’re trying to enable access to plaintiff’s work

Parody vs Satire
 Parody: commentary or criticism on the original work
o more than just “cute and funny”
o stronger claims for fair use bc often need to use the work in order to parody (tease/poke fun) at it –
commentary aspect gives it more value (first amendment)
o parodies are supposed to be funny. Doesn’t mean you actually have to find it funny – more just --
could it plausibly be conceived as funny
 Satire: commentary or criticism about something else, not the original work

60
o ex: OJ Simpson story told Dr. Seuss-style. The commentary is about OJ, and just using Dr. Seuss bc
it’s cute and recognizable
o they didn’t have to use the Dr. Seuss work, whereas 2 Live Crew parody had to use the original Roy
Orbison song “Pretty Woman”

Fair Use Attack Outline


(§107)
Notwithstanding the provisions of sections 106 and 106A, the fair use of a copyrighted work, including such
use by reproduction in copies or phonorecords or by any other means specified by that section, for purposes
such as criticism, comment, news reporting, teaching (including multiple copies for classroom use), scholarship,
or research, is not an infringement of copyright. In determining whether the use made of a work in any
particular case is a fair use the factors to be considered shall include—
 (1) the purpose and character of [defendant’s] the use, including whether such use is of a commercial nature
or is for nonprofit educational purposes;
o Commercial: question isn’t really about making money, but rather, is the defendant, by its use,
attempting to use the work without paying the customary price
 Speaks to original intent of copyright, to further progress in science and social benefits –
perhaps non-profit uses align more with this purpose
 Just bc commercial  isn’t bar to fair use
 Harper: avoiding paying customary price for this “rare” work
 Consider public benefit
(7) Sega: more independent games
(8) Perfect: Google providing public benefit
(9) HathiTrust: altho not transformative, providing access to print-disabled valid fair
use purpose
o Transformative use – does the second work add something new – a new meaning, message or
content? Or is it merely superseding the original creation? (Acuff)
 Seinfeld Trivia: not transformative, no critique/commentary – just repackaged TV show in
trivia book form
 Grateful Dead: images for historical/artefactual purposes, different than original use of
promotion/artistic expression
 Perfect 10: difference function/purpose, enabling access & directing user
 Cariou: added new meaning/message
 Prince: entirely different aesthetic, new expression (also not attempting to comment)
 Sega: ultimate work created very different than copied code
 HathiTrust: full-text search doesn’t supersede book’s original purpose, index has entirely
different purpose
 Consider how others perceives the work (not artist’s intent) (Prince)
o Parody (not satire) – they require the underlying work
 Acuff-Rose: parody adds something new and provides social benefit
o News reporting:
 Rejected in Harper bc wasn’t about reporting the facts – was about scooping
o Character of use
 Harper: bad faith, “purloined” the unpublished manuscript
 (2) the nature of the copyrighted (plaintiff’s) work;
o This factor isn’t that important where the secondary use is so transformative (Grateful Dead, Prince,
HathiTrust), or on the contrary, where a use is so minimally transformative (Seinfeld)
o Consider: some works are closer to the core of intended copyright protection than other (Acuff)
o this factor matters a LOT as far as what happens in the other factors
o Facts vs. fiction: stronger protection for fiction bc imp for society to have access to facts
61
o unpublished vs. published: stronger protection for unpublished bc copyright customarily protects
author’s decision if/when to publish (Harper)
o Function: copyright law doesn’t protect functional/efficient stuff so we need to filter that out (Sega)
 (3) the amount and substantiality of the portion [defendant] used in relation to the [plaintiff’s] copyrighted
work as a whole; and [what copier took from original work]
o Proportion of the original work used, not how much of the secondary work comprises the original
o Did they take more than necessary?
o Can’t excuse a taking by just showing that what you took was unsubstantial w/respect to the
infringing work (“no plagiarist can excuse the wrong by showing how much of his work he did not
pirate”)
o analysis looks a lot like the qualitative/quantitative substantial similarity test
 intuition = the more you take, the less “fair”
o is the extent of copying consistent with, or is it more than necessary to further the purpose and
character of the use
 Acuff: took the “heart” but this is necessary in a parody
 Prince: secondary use must be permitted to “conjure up” at least enough of the original to
fulfill its transformative purpose
o Harper: not a ton, but took the most important parts – direct quotes and the “heart”
o Seinfeld: use a lot and used lots of the best lines/bits
o Grateful Dead: altho fully produced, small size
o Sega: afford little weigh to this factor bc ultimate use of original work was limited
o Perfect: could have copied “less” but court doesn’t want to tell tech ppl how to do stuff
o HathiTrust: was necessary to copy entire works in order to enable full-text search
 Was necessary to copy/retain text and digital images for disabled access
 (4) the effect of the [defendant’s] use upon the potential market for or value of the [plaintiff’s] copyrighted
work.
o Harm that results because the secondary use is a substitute for the original work? (HathiTrust)
o Where use is so transformative, hard to presume market hard (Perfect)
o OK to damage a market (e.g., book review trashing book) but you can’t supplant it
 Where a work is transformative, market substitution is less certain
o Have to think about potential impact on the market for all works
o This can be difficult for the defendant to prove bc difficult to prove access of harm
o Does the secondary use usurp or substitute the original work’s market?
o Look to use if secondary use usurps plaintiff’s potential to develop a derivative market (Grateful
Dead)
o Copyright owners cannot preempt exploitation of transformative markets which they would not
generally develop or license others to develop
o Copyright owners cannot prevent others from entering fair use markets merely by developing or
licensing a market for parody, news reporting or other transformative uses of its own creative work
o Does the secondary work usurp the market for copyright holder’s original work and derivative
markets?
 Yes: Seinfeld: SAT substituted for a derivative market that a TV copyright owner generally
would have developed or licensed for others to develop
 No: Prince: very different audiences, no evidence Cariou would ever develop/license
secondary uses of his work in vein of Prince’s work
o Harper: Nation impeded “progress” by displacing Harper’s sales; actual market effect
o Acuff: parody and original serve different market functions
 No protective derivative market for criticism
 No evidence that derivative market for rap music was harmed
o Grateful Dead: paying license fees to others doesn’t mean can’t still use fair use
62
o Sega: disassembly of object code in Sega game was necessary
o HathiTrust: no substitution for the actual books

*remember, no trump cards here – balance the factors

The fact that a work is unpublished shall not itself bar a finding of fair use if such finding is made upon
consideration of all the above factors.

Cultural Interchange
(564)
*note the preamble – uses that further the development of a common culture: criticism, comment, news
reporting, scholarship, research – all promote the learning and the arts. These uses help to produce a public that
is educated and informed.

Focus: consent and reasonable expectation of copyright owners


Memoir about pardoning Nixon
Harper & Row v. Nation Enterprises (565)
F: Gerald Ford contracted with Harper & Row to publish his memoirs. Harper negotiated a prepublication
agreement with Time Magazine for the right to excerpt 7,500 words from Ford's account of his pardon of
Nixon. Before Time released its article, an unauthorized source provided The Nation with the unpublished Ford
manuscript. Subsequently, The Nation put together a story composed of quotes, paraphrases and facts drawn
exclusively from the manuscript (scooping Time). Nation story contained 300 words of direct
quotes/infringement. Harper sued The Nation, alleging copyright violation. Nation argued fair use.

H: Infringement, not fair use

Application of the factors


Factor 1: purpose and character of defendant’s (Nation) use
 Commercial: The Nation is commercial, they get paid for access – making money off this excerpt
o Consider: If B is making money off using A’s work, less likely A would have consented to B’s use
o tho non-profit uses aren’t always “non-commercial”. Reputation gain.
 Question isn’t whether defendant is making money, but if the defendant, by its use, is attempting to avoid
paying the customary price (what one would typically for the use/work) – user stands to profit from
exploitation of the copyrighted material without paying the customary price
o Here, unpublished, “rare” (only 1 memoir by Ford) comments by Ford
o Even more importantly, Time paid for the work, shows that this use is inappropriate because Nation
competitors seem to think they should pay for this type of use
 Character:
o Not good faith: Court seems upset about what Nation did – “purloined” – like theft. Different than if
just found it.
o Intended purpose of supplanting the copyright holder’s commercially valuable right of first
publication
 News reporting? Nation argues “news reporting” to publish what Ford thinks about his pardoning of Nixon
and that’s why they had to quote so much, bc it’s all about how Ford expressed himself about these facts)
o Court rejects this. This isn’t news reporting. Nation took something they shouldn’t have taken and
made a news event about the scoop/pre-publication. Publishing wasn’t just about the facts and
Ford’s expression of the facts. Publication was about the SCOOPING.
o No independent commentary, research or criticism
63
Factor 2: nature of the plaintiff’s (Harper) work
 Harper’s work was unpublished facts; unpublished allowed Harper to “win” this factor
 Fact vs. Fiction: Generally, more important for society to have access to facts than fiction
o May be circumstances where ppl need access to the underlying facts and it may be best to just copy
the plaintiff’s work (Hindenburg case). Would it be “risky” to not allow others to access plaintiff’s
owe their origin to the plaintiff.
 Published vs. Unpublished
o Copyright customarily protects the author’s decision to decide to publish or not, as well as when to
publish
 Sometimes, want to release at certain time of year, same time as something else, or not at
same time as something else (due to competition)
 Unpublished is factor tending to negate fair use defense
o Should defendants ever have the right to publish before the original author? Sure, maybe in politics
(ex: letter where candidate states he killed someone – publish before election)

Factor 3: amount and substantiality of what the defendant (Nation) used & the relationship between
what defendant (Nation) took and the plaintiff’s (Harper) work
 “Heart”: Nation took the most important parts. The rest of the memoir wasn’t that interesting/important.
They took the real substance/heart – the reason ppl were buying the book in the first place.
o Nation sort of arguing the same thing ^ -- they took the part that’s newsworthy…the
o Remember, a taking may not be excused merely because it is unsubstantial with respect to the
infringing work (“no plagiarist can excuse the wrong by showing how much of his work he did not
pirate”
 Direct quotes: Nation took a lot of direct quotes. Argued it had to bc what mattered here were the precise
words the Ford said – not the fact that he pardoned Nixon.
o But court says they used too many direct quotes of the “heart” of the material
o Where a substantial portion of the infringing work was copied verbatim – evidence of qualitative
value of the copied material – both to the originator and the plagiarist

Factor 4: effect on the market for the plaintiff’s (Harper’s) work


 Court says this is the most important factor
 Not just potential, but actual market effect (Time cancelled its projected publication and refused to pay)
 We care about whether Harper can reap the appropriate economic rewards from its creativity – the creativity
we appreciated when we granted Ford/Harper the copyright (we want to incentivize!)
 So where Nation displaces Harper’s sales, this impedes, and doesn’t promote progress (doesn’t incentivize
Ford to write a memoir)
 Nation directly competed for a share of the market for prepublication excerpts
 Are more or less ppl going to read the memoir after Nation publishes the excerpt?
o More argument: all attention is good – going to drive ppl to the memoir. And now Nation readers are
learning about Ford’s thoughts on the pardon – and they wouldn’t have learned about it from Time
o Less argument: what they published was the heart of the book. Ppl will read the excerpt and won’t
need to buy the book.
 To counter, TIME was going to publish an excerpt anyway and Harper/Ford was clearly OK
with that/thought it was worth it!
 Remember, have to think about the potential impact on all the market for all works. Derivative works, movie
studio biopic, etc.
o Like factor 1, the court is thinking about the customary markets that authors are allowed/not allowed
(“competitively) to enter exclusively
64
o If it impedes these customary markets, then not fair use. Tho, this is circuitous…it’s not fair use if it
is and it’s not fair use if it’s not

Transformative Use
Critiquing/criticizing the original work
Pretty Woman
Campbell v. Acuff-Rose Music (573)
F: 2 Live Crew parodied the Roy Orbison/William Dees song "Oh, Pretty Woman" in a song. Acuff-Rose
Music (which owns the composition rights the Orbison song) sued for copyright infringement. Campbell is
member of 2 Live Crew.

Note on music copyright law: If 2 Live Crew had wanted to just create a cover, they could have just gone to
Harry Fox to get a mechanical license. But they didn’t want to just create a (rap) cover of the song. They
wanted to create a derivative work (keep the basic structure and some lyrics, but make lots of changes,
including changing some lyrics) – compulsory mechanical license doesn’t cover that. They asked Acuff-Rose
for permission, who refused, put produced/published anyway.

Lower court: not fair use – presumption that when defendant’s use is commercial, this is not fair use

H: fair use.

Application of the factors


Factor 1: purpose and character of 2 Live Crew’s use
 In regards to lower court discussion, we never meant for one factor to overtake all the other factors. It’s
supposed to be a balancing of the issues, no single factor trump.
 2 Live Crew’s commercial use certainly doesn’t help its case, but doesn’t necessarily kill it
o Some commercial uses can still constitute fair use (ex: newspapers)
 Transformative: does 2 Live Crew’s work merely substitute/supersede Orbison’s work, or does it add
something new? A new meaning, message, or content? Yes – parody provides social benefit.
o 2 Live Crew song comments on/criticizes the original work, commentary on society as a whole
 this is the central portion of the court’s fair use analysis (links factors 1 and 4)
 tied into goals of copyright/”progress”
 head-scratching bc statute doesn’t mention “transformation” – that word comes up in derivative works
definition tho. So, court seems to be saying that some sorts of transformations are the right transformations
o transformation doesn’t necessarily equal fair use, or derivative work. Rather, some kinds of
transformation will constitute infringement and some will not.
 So what kind of transformation is 2 Live Crew’s work?
o Parody – a commentary or criticism
 Not satire. Parodies have to use the original/underlying work – have stronger fair use claims
than satire

Factor 2: the nature of Roy Orbison’s work


 Not really discussed here…just says that this factor calls for recognition that some works are closer to the
core of intended copyright protection than others – with the consequence that fair use is more difficult to
establish when these works are copied

Factor 3: the amount/substantiality of the portion 2 Live Crew used in relation to Orbison’s work
 Quantity and value of the materials used were reasonable in relation to the purpose – parody
65
 Even if they took the “heart” (opening riff and first line), this “heart” us what most readily conjures up the
song for parody, and it is the heart at which parody takes aim
 Of course, can’t just call something parody to get away with copying – must look at context – here, they
copied the first line of the original and then departed markedly with different lyrics and sounds

Factor 4: the effect of 2 Live Crew’s use on the potential market/value of Orbison’s work
 Where commercial use amounts to mere duplication of the entirety of an original, it clearly supersedes the
original – making it likely cognizable market harm to the original will occur (is market replacement)
 But where the second use is transformative, market substitution is less certain and market harm won’t be so
readily inferred
o Parody and the original usually serve different market functions
 Copyright law must distinguish between biting criticism which merely suppresses demand and infringement
– which usurps it
 You can damage the market (e.g., write a terrible review of a book), but you can’t supplant the market
 Rule: There is no protectible derivative market for criticism
o The market for potential derivative uses includes only those that creators of the original works
would, in general, develop or license others to develop
o The unlikelihood that creators of works will licensed critical reviews or lampoons of their own
productions removes such uses from the very notion of the potential licensing market
o Here, can look at the derivative market for rap music, consider market substitutions
 No evidence that a potential rap market was harmed by Crew’s parody rap version
 note: Court calls fair use an affirmative defense, although the statute says “not infringement of copyright”
o court is reading statute as affirmative defense, so defendant has burden of proof
o pretty difficult for defendant to prove this factor – that 2 Live Crew’s use didn’t harm
market/value of Orbison’s work

Fair use question is now: is the work transformative in the right way, or does it merely supplant the plaintiff’s
work – infringement?

Superseded original work’s market


Seinfeld Trivia Book
Castle Rock v. Carol Publishing (582)
F: "The SAT" - Seinfeld trivia book (published by Carol). Castle Rock (Seinfeld) sues.

H: Not fair use. The book is a derivative work, and thus, infringing. Court doesn’t find much new
meaning/content in what SAT did. SAT just using the entertaining aspects of Seinfeld – which seems to be
within original author’s right.

Application of the factors


Factor 1: the purpose/character of SAT’s use
 Per Campbell, SAT needs to prove the work for transformative – that is doesn’t simply supersede the
original work – it adds additional content/message
o SAT argues that it’s a companion work, adding stuff. The book transform the experience of
watching Seinfeld. Like Campbell, it’s a critical commentary/critique of the underlying Seinfeld text
 Court disagrees. SAT book basically just free-riding on Seinfeld. Book even says it exists to
give you your Seinfeld “fix” in between episodes! Not transformative – fitting the same
market. Book repackages Seinfeld, no commentary, criticism, parody, research etc.

66
 Altho a secondary work doesn’t necessarily need to transform the original work’s expression
to have a transformative purpose, SAT so minimally alters Seinfeld’s express – further
evidence of book’s lack of transformative purpose
 This book is something the plaintiff could have done.
 But does it matter that the plaintiff didn’t do this? Will discuss more in 4th
factor…but for now, remember Harper  didn’t matter that that it was going to be
published anyway, bc the plaintiff has the exclusive right to decide where/when it’s
published. And THIS isn’t about published vs. unpublished. It’s a derivative work
and the original author has the right to decide if this work should even exist or not
(since original author can decide what segments of the market to enter or not).
 Consider: what could SAT have done differently to improve its chances on this factor?
o Maybe publish a chunk of text and then ask a “discussion” question – encouraging ppl to
think/critique the work in an interesting way.
 The criticism market is not one the original author is typically given the exclusive right to
enter

Factor 2: the nature of Seinfeld (favors plaintiff)


 Court doesn’t pay much attention to this factor since the SAT is so minimally transformative

Factor 3: the amount/substantiality of the portion SAT used in relation to Seinfeld (favors plaintiff)
 Inquiry: whether the extent of copying is consistent with, or is more than necessary to further the purpose
and character of the use (Acuff)
 SAT used a lot of Seinfeld! And seems like they used a lot of the best lines/bits

Factor 4: the effect of SAT upon the potential market/value of Seinfeld


 Inquiry: does the secondary use usurp or substitute the market for the original work (Acuff)
 SAT substitutes for a derivative market that a TV copyright owner like Castle Rock generally would
develop or license for others to develop

Hypos/Other Works
 The Wind Done Gone (“Gone with the Wind” book told from perspective of slave): parody – comments
on/criticizes a prior work  fair use
 Come Through the Rye (unauthorized sequel of “Catcher in the Rye”): if change the character in the “right”
way, then fair use. But if don’t use character in the “right” way – infringes
 What about “Catcher in the Rye – Zombies”? Fact-intensive inquiry.
o If using a lot of the same words in large chunks and just adding random zombies  problematic
o But if changing a lot, say, incorporating the zombies in a meaningful way  less problematic
o Consider market/audience – do ppl still want to read the original?

Fair use
Grateful Dead coffee table book
Bill Graham v. Dorling Kindersley (586)
F: Dorling Kindersley published a Grateful Dead coffee table book. Bill Graham claimed he owns 7 of the
images displayed in the book (images had been on event posters and tickets – created as ads for the concerts).

H: fair use

Factor 1: purpose and character of coffee table book’s use


67
 Courts frequently afford fair use protection for copyrighted materials in historical scholarship
 Coffee table book using the images for historical/artefactual purposes – give readers a sense/feel of the
culture. Different than posters original use – to get ppl to go to concerts (promotion and artistic expression)
o But maybe the purpose of the original poster was not just as an ad, but also as art/artifact – say, put
up on a wall in your room. Maybe the purpose of the design of the poster was just to look cool.
o Also, consider that the poster isn’t receiving any commentary/criticism in the book – it’s just playing
an entertainment role (same role as the original work)
 Also note that DK significantly reduced the size of the reproductions, combining them with a timeline, texts,
original graphics – creating a collage of text and images. The images at issue used to enrich the presentation
of the cultural history of the Grateful Dead (not to exploit copyrighted artwork for commercial gain)
 Also, the images are inconsequential portion of DK’s book – less than 1/5 of 1%

Factor 2: Nature of original posters


 Factor not as important when the copyrighted work of art is being used for a transformative purpose

Factor 3: Amount/substantiality of coffee table book used in relation to original posters


 The entire thing is produced, tho is smaller size so lots of detail washed out -- visual impact of the artistic
expression is significantly limited

Factor 4: Effect of coffee table book’s use upon potential market or value of original posters
 No claim that DK’s use of images impacted plaintiff’s primary market for sale of the poster images; rather.
Court looks to whether DK’s use usurps plaintiff’s potential to develop a derivative market
o DK’s use transformative, and a copyright holder cannot prevent others from entering fair use
markets merely by developing or licensing a market for parody, news reporting or other
transformative uses if its own creative work
 Facts that DK paid license fees to others, and even tried to pay plaintiff (couldn’t agree on price) doesn’t
establish that DK can’t, in the alternative, make fair use of those images
 Does the coffee table book undermine the plaintiff’s incentive to create the work in the first place –
essentially – if defendant and others can get away with what they did – will original authors still feel
incentivized to create stuff like this?
o Seems like defendant’s work doesn’t affect plaintiff’s work/licensing

Consider
-what else is on the page? On one page is a (licensed) photo of Jerry Garcia. So…shouldn’t they also have
licensed the art? Isn’t actually more important to license creative art, rather than a photo that just shows how
Jerry Garcia actually looks?

Consider
-what if the coffee table book was a book of posters? Likely a losing case!
-what if the coffee table book helps the plaintiff?

Appropriation Art
(591)
*definition: “the more or less direct taking over into a work of art a real object or even an existing work of art”

Koons
-famous appropriation artist
-lost fair use argument in puppies case
68
-won in feet case – made statement in deposition about how he wanted to add something new, new insights, etc.
(sound like his lawyer told him to talk “fair use”!)

Add new expression & different market (not just superseding original work)
Rastafarian Photos
Cariou v. Prince (591)
F: Prince used Cariou’s photos of the Rastafarian community in Jamaica (cut from Cariou's book) in his own
art. In some of Prince’s art, he increased the size of Cariou’s photos, blurred or sharpened them, added
additional content, sometimes compositing multiple photos together or with other works. Prince had a show of
these photos at the Gagosian gallery. Prior to this show, a gallery owner contacted Cariou about exhibiting his
work in NYC. The gallery owner then learned of the Prince Gagosian show and decided not to go forward with
exhibiting Cario’s work, mistakenly thinking that Cariou was involved with the Gagosian Prince show. Cariou
sues.

Deposition: Prince says there’s no new meaning or message in his work – he’s not commenting/critiquing
Cariou’s work – he just “makes things up.”

H: all but 5 of the photos were fair use. They manifest an entirely different aesthetic from Cariou’s photographs

Application of the factors


Factor 1: new work must alter the original with “new expression, meaning, or message” – it is transformative.
 Work doesn’t have to comment on the original or its author in order to be considered transformative
 Prince’s works manifest an entirely different aesthetic from Cariou’s
o Cariou’s black and white photos are serene and deliberately composed portraits and landscape
photos depicting the natural beauty of Rastafarian and their surrounding environs
o Prince’s art are crude and jarring works – hectic and provocative. They are color collages with
distorted humans and other forms and settings, and measure between ten and nearly a hundred times
the size of the photos. His composition, presentation, scale , color palette and media are
fundamentally different and new compared to the photos, as is the expressive nature of Prince’s
work.
 Analysis moving from intent to how the audience perceives works
o How the work in question appears to the reasonable observer, not simply what an artist might say
about his work
 Lower court was wrong to say that to qualify got fair use defense, the secondary work must comment on or
critique the original work. This is not required. Prince’s work is transformative even without commenting
on Cariou’s work or on culture.

Factor 2: nature of the copyrighted work (recognizing that some works are closer to the core of intended
copyright protection than others, with the consequence that fair use is more difficult to establish when the
former works are copied)
 Cariou’s work is creative and published, weighing against a fair use determination
o However, just as with the commercial character of Prince’s work, this factor may be of limited
usefulness where, as here, the creative work of art is being used for a transformative purpose
o Work is very transformative

Factor 3: amount/substantiality of the portion used in relation to the original work


 Consider the proportion of the original work used – not how much of the secondary work comprises the
original

69
 Consider not only the quantity of the materials taken but also their quality and importance to the original
work
 The secondary use must be permitted to “conjure up” at least enough of the original to fulfill its
transformative purpose
 In 25 of his artworks, Prince transformed the photos into something new and different – factor weighing
heavily in Prince’s favor

Factor 4: effect of the secondary use upon the potential market for the value of the copyrighted work
 Focus is not on potential damage to Cariou’s market or derivative markets, but rather if the secondary works
usurps the market of Cariou’s original work. Prince did not usurp the market for Cariou’s photos (neither
primary or derivative markets)
o An accused infringer has usurped the copyrighted work’s market (including derivative market)
where the infringer’s target audience and nature of the infringing consent is the same as the original
(Seinfeld) – not the case here
 The more transformative the secondary use, the less likely the secondary use substitutes for
the original
o Prince’s audience is very different than Cariou’s, appeals to a very different sort of collector
o Nothing in the record suggests Cariou would ever develop or license secondary uses of his work in
the vein of Prince’s artwork
o Carious didn’t aggressively market his work, and he earned just over $8,000 in royalties from the
photo book
o Nothing in the record suggests that anyone will not now purchase Cariou’s work, or derivative non-
transformative works (either Cariou’s own or licensed by him) as a result of the market space
Prince’s work has taken up
 Lower court wrong to focus principally on the question of damage to Cariou’s derivative market – the
question is not whether the secondary use suppresses or even destroys the market for the original work or its
potential derivatives, but whether the secondary use usurps the market of the original work
 An accused infringer has usurped the market for the copyrighted work (including the derivative market),
where the infringer’s target audience and the nature of the infringing content is the same as the original.
Prince’s works are in a market that Carious would not have entered
o (ex: Seinfeld SAT book usurped the show’s market bc the trivia book substituted for a derivative
market that a TV program copyright owner would generally develop or license others to develop.
Seinfeld, if it wanted to, could have produced this work)
o Prof: If Cariou did what Prince did – put the lozenges and guitar, etc. – would sell for $0. This is
about Prince.

Technology
(602)

General
 Different analysis in tech
 Technology cases think about fair use differently – the nature of what plaintiff did and how what the
defendant is doing is different than what plaintiff did

Disassembly
*decompilation necessarily involves making copies of large portions of the copyrighted work

70
Disassembly is fair use
Sega game code
Sega v. Accolade (602)
F: Video game publisher wants to create games that can be played on Sega Genesis. So they took a Sega game
and copied it in order to figure out which parts of the game were interacting with the Sega console. They
weren’t copying because they wanted to copy the game – this was just about the code that interacts with the
console. They needed the code in order to build it into their own games. Sega sued.

Accolade “reverse engineered” Sega’s video game programs in order to discover the requirements for
compatibility with the Genesis console. Then, it created a development manual that incorporated the info it had
discovered about the requirements for compatibility. Then it created its own games. Didn’t copy Sega games,
just relied on the info it had collected about interface specifications

H: fair use. Accolade’s game was legitimately different than Sega’s game. They only copied in order to create
their own game. Generally, copying is OK if you’re only copying in order to get to your own final result –
another thing.

Note: if a work is largely functional, it receives work protection under copyright act. Fact that computer
programs are distributed for public use in object code often precludes public access to the ideas and functional
concepts in these programs, conferring on copyright owners a de facto monopoly over those ideas and
functional concepts – defeating the fundamental purpose of the Copyright Act

Application of the factors


Factor 1: nature & purpose of Accolade game/ transformative?
 Transformative: (depends on whether talking about the code or the game)
o Argument for: the ultimate work that will be produced due to the copying will look very different
than the game that was copied
o Argument against: Accolade took a Sega video game and created a video game for Sega.
 Consider: are the two games similar/competing with each other?
 Also, Accolade took ALL the code!
 Commercial: Yes, but Accolade didn’t seek to avoid paying a customarily charged fee nor simply Sega’s
code – it wrote its own procedures based on what it learned through disassembly
o Accolade’s ultimate purpose was to release its own Sega-compatible games
o Also consider public benefit, Accolade’s identification of the Genesis compatibility functional
requirements has resulted in an increase in the number of independently designed video games
offered for use with Genesis

Factor 2: Nature of Sega game (this factor tell us how to weigh the additional factors)
 The Sega game software looks like instructions – is functional (like the bookkeeping book in Baker v.
Selden!) Per Altai, have to filter out all the functional/efficient stuff first. Now that we’ve filtered that out,
how much of the software is actually expressive, and thus copyrightable in its own right, not much!
 This factor tells us that factor 3 doesn’t matter that much bc of the why – WHY Accolade used the Sega
game – and here – Accolade only used the game to get to the unprotectable stuff (functional code)
 Note: by copying the game, they got both the functional and expressive aspects. This is different than Baker
v. Selden bc while the systems/forms were unprotectable, the introductory essay was. So could couldn’t
copy that. But here, the only WAY to copy the functional unprotected stuff was to also, “accidently” copy
the expressive stuff

Factor 3: Amount/substantiality of Accolade game used in relation to Sega game

71
 Copied everything – all the code
 Afford this factor little weight tho bc the ultimate use was limited

Factor 4: Effect of Sega game upon potential market or value of Sega game
 The Accolade game is a competitor with Sega’s games – is at least a partial substitute
 Tho, ppl may buy both video games! Ppl generally buy more than one game!
 Also, the Accolade game is supplemental to Sega’s system (played on the system)
 Disassembly of the object code in Sega’s video game cartridge was necessary in order to understand the
functional requirements for Genesis compatibility – disassembly of object code necessarily entails copying
o Sega games contain unprotected elements which could not be examined without copying
 Does it matter that Sega was charging licensing fees for the materials Accolade copied?
 Does it matter that Sega was very controlling about anything associated with Sega? (whereas Nintendo lets
any companies make compatible games, like today’s comparison of Apple v. Microsoft)
o Economic situation of razor and blades. Sega prices the console fairly cheaply, knowing ppl will
have to buy the fairly expensive games. Now, Sega may have to lower the games prices to be
more competitive if companies like Accolade are creating games which are compatible with its
console system! …should copyright law care about this?
 Remember, Duke Nukem – should other ppl be able to create “sequels”?

Remember Google v. Oracle


API - enabled coding in 1 language (Java) to interact w a bunch of other programming languages. And google
came along and used that code.
-recently, google won in a fair use trial
-likely argued that they used it mostly for their functionality, they don't care about the expressive bits
(they're just by-products) -- just like the Sega case
-courts are sensitive to these situations, where someone is using copyrighted work to try to prevent competition
(in a non-copyright way -- ex: of printer toners putting an original bit of code on the toner to try to prevent ppl
from copying)
-copyright law should not be used for this no -- makes courts unhappy

Online Search
Google using Perfect’s stuff not for entertainment value, but just to “direct”
Porn thumbnails
Perfect 10 v. Amazon (609) (exploring defendant's fair use argument)
See earlier factor. Recall: (1) the thumbnails on the google server were a prima facie violation – reproduction
and display; (2) the bigger photos weren’t actually on Google (were on new/other page), not a violation

H: Google’s thumbnail photos are fair use. Google using Perfect’s work not to compete, but to enable access to
Perfect’s work

Application of the factors


Factor 1: purpose/character of work  highly transformative (weighing in favor of Google)
 Google took a big image and made it little
 Highly transformative
o Search engine transforms the image into a pointed directing a user to a source of information –
electronic reference tool
72
o Google uses the images in a new context to serve a different purpose
o Even MORE than that, they transformed the function of the image. The original has as
entertainment purpose/value. Google’s function/value is to point/tell the user where something
is!
o Not giving you the actual thing, just telling you WHERE to find the thing
o Could Google just say “the image is here” with a link? That’s not efficient! Ppl don’t want that.
Ppl like the thumbnails, then they know what the image is going to be
 Also, consider public benefit, outweighs Google’s commercial and superseding use

Factor 2: nature of the copyrighted worked (weighing slightly in favor of Perfect 10 bc previously published)

Factor 3: amount/substantiality of the portion used (favors neither party)


 Google copied all of the work
 Could they have copied less? Sure, but they wouldn’t want to.
 Could they have blurred or cropped the images? Sure, but that’s less efficient bc ppl won’t know if the
photo is the one they’re looking for. Or they may think the blurred/cropped version is what the full size
image will look like.
 Court seems to now want to tell tech ppl how to do stuff…doesn’t want to tell Google “redistribute or blur
20% of the image”
o Also, due to Google’s scale, it would be pretty impossible for Google to go through every image
and do that

Factor 4: effect on market (favors neither party)


 Bc Google’s use is highly transformative, market harm cannot be presumed
 What about Perfect 10 creating reduced-sized images for cellphones? Thumbnails will totally supplant this!
o Court: there’s no proven market for this
o Sure, there could have been a real, cognizable harm if you were having ppl pay for reduced size
images – but you’re not actually doing this
o But because fair use is an affirmative defense, doesn’t Google have to prove there wouldn’t be a
market for the reduced-sized images? …doesn’t seem to happen here
 No evidence of usurping the cell phone image market
 And, now that we know this is fair use, it’s not going to BE a market harm in the future

Access to Knowledge
Digital libraries
Authors Guild v. HathiTrust (613) (providing stuff to disabled ppl)
F: HathiTrust scans and compiles complete published works for 3 purposes/uses. (1) search term function (not
providing full text, just pages numbers); (2) disability access function, reproducing works for individuals with
disabilities; (3) copying and replacing lost or destroyed versions for member libraries. They got the published
works from four university libraries, didn’t ask permission.

H: fair use

Application of the factors regarding full-text search


Factor 1: purpose/character of work – transformative?
 HathiTrust’s use is transformative, what the user gets/sees is nothing like the actual books

73
o Result of word search is different in purpose, character, expression, meaning and message from
the page (book) from which it is drawn
o Doesn’t supersede the books’ original purpose
 They’re not copying the books bc they care about the actual copyrightable expressions/content in the book,
they’re copying them so that ppl can use HathiTrust’s index as a resource – so researchers know if they
should actually go out and get this book
 Court not comparing defendant’s actual copies, rather, comparing the user’s end result  just a list of words
and pages (result of using an index) which is NOT like a book! Not a substitute for the book.
o Doing something entirely different. Not adding new meaning/message, but adding an entirely
different value.
 Think: Harry Potter Lexicon case – not an infringing derivative work bc didn’t substitute
for the underlying work. Problem in that case was reproduction right violation for taking
too much

Factor 2: nature of the copyrighted work


 Factor of limited usefulness where the creative work is being used for a transformative purpose

Factor 3: amount/substantiality of the portion used


 Question: did they take more than necessary?
o No, was necessary to make use of entire works in order to enable he full-text search function
o Necessary to back up all the servers for backup purposes
 HathiTrust copies all of the books so that their search terms would be comprehensive – trying to produce
something like a library or index, so needed to copy everything in order to see where the particular search
terms appear
 Ppl use HathiTrust search in order to see how often a word appears in a work helpful for
researchers/scholars

Factor 4: effect on market (not binary, is about the degree of harm)


 Only concern: harm that results bc the secondary use serves as a substitute for the original work
 No evidence of economic injury by this transformative use bc by definition, transformative uses do not
serve as substitutes for the original work
 Full text search function doesn’t serve as substitute for the books
 In fact, what HathiTrust is going might be good for the books bc now ppl will learn about books they might
otherwise not have known about
 Remember, since affirmative defense, defendant has to prove no market harm

Application of the factors regarding access to print-disabled


Factor 1: purpose/character of work – transformative?
 Altho appears to be creative derivative works (over which original author ordinarily maintains control),
providing access to print-disabled is still a valid purpose altho not transformative
o Supreme court and Congress have made this clear, plus ADA

Factor 2: nature of the copyrighted work (weighs against fair use)

Factor 3: amount/substantiality of the portion used


 Needed to copy both text files and digital images – was reasonable

Factor 4: effect on market (not binary, is about the degree of harm)


 Market for books accessible to handicapped is insignificant, very minute percentage of the world’s books
74
Discussion of disability use (another of HathiTrust’s functions)
 This is a translation, violating derivative right – now braille ppl don’t have to pay for these works
 Congress seems to care about visually impaired a lot tho (ADA)
 And it seems like there isn’t a lot of entry in the Braille market
 What if it translated to Navajo (not a language book publishers often translate into)?

Google books
Something about snippets and something about a few pages
 -google is letting you see a snippet -- court said this is fair use. They can go beyond what hathiTrust did.

***outcomes are largely determined by how court likes google. And court seems to like this. If this didn't
constitute fair use, then it wouldn't happen! Google would have to get licenses from everyone - go to each
author or publisher - and interact with each one of these ppl and that's too much.
-and the transaction costs alone involved in something like that would just be so high/prohibitive
(then again, can't google afford it?)
-but other companies can't afford it. And we want competition!
-we know Google wouldn't go through all this if he had to license/pay each. So the fair use
doctrine actually allows things of valuable use to come into existence that wouldn't have come
in otherwise!

*it has to be a special kind of value. The right kind of awesome that we need fair use to bring about.

HARPER - they took out all the bad parts, showed you the good parts
-this here, is different, the Google books snippets or Perfect 10 thumbnails are pointing you toward the full
book or to the Perfect 10 image.

Google only works bc it's big. A search website that only searches 20% isn't a successful site. And the only way
it can get this big is through fair use.

Betamax
Sony Corp. v. Universal (623)
F: Sony manufactured and sold the "Betamax" home video tape recorder (forerunner/competitor of VHS/VCR).
Universal owned the copyrights to television programs broadcast on public airwaves. Universal sued Sony for
copyright infringement, alleging that because consumers used Sony's Betamax to record Universal's copyrighted
works, Sony was liable for the copyright infringement allegedly committed by those consumers in violation of
the Copyright Act.

Note: we’re ignoring secondary liability – ppl using Sony’s equipment for these illicit uses

H: Betamax is capable of substantial non-infringing uses – fair use and no copyright infringement.

Two concerns about Betamax


1) Ppl recording shows and “building libraries” so they don’t have to buy VHS
a. This is obviously not fair use – just a substitute for buying the VHS/going to blockbuster

75
2) Time-shifting: Ppl recording shows bc they weren’t home to watch the show. Then after you watch it, they
erase it and use the tape for something else. So this this fair use?
a. PBS, Mr. Rodgers say: time-shifting is fine! Educational use. Consent! They approve of this
practice, think it’s valuable.
b. NBC, CBS obviously disagree…so who’s interest trumps?

-significant likelihood that a substantial number of copyright holders who license their works for free public
broadcasts would not object to having their broadcasts time-shifted by private viewers
-Universal failed to show that time-shifting would cause non-minimal harm to the potential market for, or the
value of, their copyrighted works.

*think about how technological advances may affect fair use


-technologists being very careful to build their technology so that it falls into fair use
-courts challenge is about letting technology help ppl in helpful ways

…is ad-skipping a violation. Hurting TV producers. Does copyright law care about this kind of harm?
Copyright law didn't care about what Sega did. Fair use doctrine trying to figure out which kinds of harm it
should be concerned with

Question: 107 prologue and factors relationship


-courts care most about the factors
-if the factors implicate one of those typical fair uses, then that matters to them
-but Sony, time-shifting…none of these relate to the prologue (criticism, comment) but court says that's still
fine. Is still fair use.

76

You might also like